krok 2 - testcentr.org.ua · d. monoblastic leukemia e. acute herpetic stomatitis 18. a 57-year-old...

37
Sample test questions Krok 2 Stomatology ()

Upload: vanmien

Post on 08-Jul-2019

289 views

Category:

Documents


1 download

TRANSCRIPT

Page 1: Krok 2 - testcentr.org.ua · D. Monoblastic leukemia E. Acute herpetic stomatitis 18. A 57-year-old retired man complains of attacks of burning pain and rashes on the skin of his

Sample test questions

Krok 2 Stomatology

()

Page 2: Krok 2 - testcentr.org.ua · D. Monoblastic leukemia E. Acute herpetic stomatitis 18. A 57-year-old retired man complains of attacks of burning pain and rashes on the skin of his

Терапевтична стоматологiя 2

1. A 48-year-old man complains of itchinggums. Objectively: the gums are firm; cold watercauses short-time pain in the teeth. X-ray imagingshows osteosclerosis of the alveolar septum:close-meshed bone structure, the alveolar septumheight and intactness of the cortical plates isretained. What is the most likely diagnosis?

A. Initial periodontosisB. Atrophic gingivitisC. Initial periodontitisD. Periodontitis, stage IE. Periodontosis, stage I

2. A 55-year-old man complains of heartburn andunpleasant sensations in his tongue. The patienthas been suffering from gastritis with low acidityfor approximately 5 years. What changes in thetongue are the most likely to be detected in thispatient?

A. Atrophied and smoothed out lingual papillaeB. Hypertrophic lingual papillaeC. Erosions on the lateral surfaces of the tongueD. Coated tongueE. Fissured tongue

3. A 25-year-old patient complains of profusegingival hemorrhages, pain in the oral cavity,weakness, fatigue, fever up to 38oC. Thesepresentations appeared a week ago. Objectively:the patient is pale, adynamic. Examination ofthe oral mucosa reveals multiple hemorrhages,friable gums, necrotic areas on the tops ofgingival papillae, as well as enlarged, soft,painless lymph nodes. The oral mucosal lesioncan be a symptom of the following disease:

A. Acute leukemiaB. Chronic leukemiaC. Vincent’s stomatitisD. Hypovitaminosis CE. Intoxication with heavy metal salts

4. During preventive examination a 40-year-old man presents with the following changes:marginal gingiva is enlarged, torus-shaped,cyanotic, slightly bleeding when touched with adental probe; there is no pain. Staining the gumswith Lugol’s iodine solution results in light-browncoloring of mucosa. Make the diagnosis:

A. Chronic catarrhal gingivitisB. Acute catarrhal gingivitisC. Exacerbation of chronic catarrhal gingivitisD. Chronic hypertrophic gingivitisE. Generalized periodontitis

5. Having recovered from acute respiratorydisease, a patient has made an appointment witha dentist. The patient complains of pain in thegums, foul breath, inability to take food, generalweakness. Objectively: the gums are hyperemic,swollen, covered with necrotic dirty-gray coating;the gums bleed when the coating is removed.Microbiological study of tissues revealed agreat number of cocci, bacilli, fusobacteria,and spirochaete. Specify the drug for etiotropictreatment:

A. MetronidazoleB. GalascorbinumC. Potassium permanganateD. TripsinE. Carotolinum (Betacarotene)

6. A 35-year-old patient complains of itch,burning and edema of lips. These presentationsoccured a week ago. Objectively: there isreddening of the red border and skin, especiallyin the area of the mouth corners, there arealso vesicles, scabs, small cracks against thebackground of erythematous affection of the redborder. What is the most likely diagnosis?

A. Acute eczematous cheilitisB. Multiform exudative erythemaC. Acute herpetic cheilitisD. Allergic contact cheilitisE. Exudative form of exfoliative cheilitis

7. A 30-year-old patient complains of a toothachecaused by hot and cold stimuli. The painirradiates to the ear and temple. Previouslythis tooth presented with spontaneous nocturnaltoothache. Objectively: on the occlusal surfaceof the 37 tooth there is a deep carious cavitycommunicating at one point with the tooth cavity.Probing at the communication point, as well ascold stimulus, causes acute pain. The pain persistsfor a long time. Electric pulp test result is 5microamperes. What is the most likely diagnosis?

A. Exacerbation of chronic pulpitisB. Acute diffuse pulpitisC. Exacerbation of chronic periodontitisD. Chronic concrementous pulpitisE. Acute suppurative pulpitis

8. A 52-year-old woman complains of periodicalappearance of a gingival fistula in the area of the15 tooth. The tooth had been treated 1,5 yearsago for caries. Objectively: the 15 tooth is filled.In the root apex projection there is a fistula;purulent exudate discharges on pressure. Toothpercussion is painless. On X-ray: the root canal isnot filled, there is a destruction focus with blurredmargins near the root. Make the diagnosis:

A. Chronic granulating periodontitisB. Exacerbation of chronic granulatingperiodontitisC. Chronic fibrous periodontitisD. Periapical cystE. Chronic granulomatous periodontitis

9. A 25-year-old man complains of genelalmalaise, high body temperature, acute gingivalbleeding, and gingival enlargement. He has ahistory of nosebleeds. Objectively the patientpresents with systemic lymphoid hyperplasia,pallor of skin and mucosa, II-III degreehyperplasia of the gingival mucosa, hemorrhagesinto the buccal mucosa, and ulcers covered withgray deposit. What examination method wouldbe optimal for diagnosis-making in this case?

A. Complete blood test panelB. Yasynsky testC. BacterioscopyD. ImmunoassayE. Blood glucose test

10. A 19-year-old young man complains of

Page 3: Krok 2 - testcentr.org.ua · D. Monoblastic leukemia E. Acute herpetic stomatitis 18. A 57-year-old retired man complains of attacks of burning pain and rashes on the skin of his

Терапевтична стоматологiя 3

constant pain in tooth 22, which intensifies onbiting with this tooth, sensation of ”protruding”tooth, and upper lip edema. The patient hashistory of upper jaw trauma. Objectively tooth22 is intact. Vertical percussion is acutely painful.The upper lip is swollen, mucogingival fold in thearea of tooth 22 is red and painful on palpation.What examination method is necessary fordiagnosis-making in this case?

A. X-rayB. Dental pulp testC. RheodentographyD. TransilluminationE. Thermometry

11. A patient complains of carious cavity in tooth11. The filling was lost one week ago. The toothcrown is dark, there is residual filling materialat the bottom of the carious cavity. Verticalpercussion is painless. X-ray shows an oval areaof bone tissue resorption with clear margins,0.4х0.3 cm in size. The root canal is filled by 2/3of its length. What is the most likely diagnosis?

A. Chronic granulomatous periodontitisB. Chronic fibrous periodontitisC. Chronic granulating periodontitisD. Radicular cystE. Exacerbation of chronic periodontitis

12. A 28-year-old man is referred for oral cavitysanation. On examination there is a filling onthe masticatory surface of tooth 17, percussion ispainless. Mucosa in the root apex projection of17 is cyanotic, vasoparesis symptom is positive.X-ray shows foci of bone tissue destruction withfuzzy margins in the area of root apices, rootcanals are not filled. What is the most likelydiagnosis?

A. Chronic granulating periodontitisB. Chronic fibrous periodontitisC. Chronic granulomatous periodontitisD. Radicular cystE. Chronic fibrous pulpitis

13. A 30-year-old woman complains of mildburning sensation in her lower lip and its dryness.She peels skin scales off with her teeth. Shehas been presenting with this condition for 10years. On examination the skin scales are grayand located on the lip from the Klein’s line tothe center of the vermillion border from angle toangle of the mouth. The scales are firmly attachedin the center and are loose on the periphery. Theirforcible removal does not result in erosions. Whatis the most likely diagnosis?

A. Exfoliative cheilitisB. Lupus erythematosusC. Meteorological cheilitisD. Allergic contact cheilitisE. Eczematous cheilitis

14. A 32-year-old patient presents with bodytemperature of 38.9oC, general fatigue, impairedspeech, inability to eat. This condition has beenrecurring for the last 4 years in autumn andspring. There are vesicles and erosions withgrayish fibrinous coating on the hyperemic andswollen labial and buccal mucosa. Nikolsky’s signis negative. What is the most likely diagnosis?

A. Erythema multiforme exudativumB. Pemphigus vulgarisC. Acute herpetic stomatitisD. Nonacantholytic pemphigusE. Dermatitis herpetiformis (Duhring’s disease)

15. A 28-year-old woman complains of persistingpain in tooth 34, which intensifies on biting.Four days ago arsenic paste was left in the34. The patient missed her appointment withthe dentist. Objective examination detectedocclusive dressing on the distal masticatorysurface of tooth 34, percussion is acutely painful.What treatment tactics would be the mostadvisable in this case?

A. Arsenic antidote is placed into the root canalunder occlusive dressingB. Dentin dressing is removed, electrophoresiswith antidote along the mucogingival fold isprescribedC. The root canal is lavaged with antidote, thetooth remains uncoveredD. The root canal is lavaged with antidote andfilledE. -

16. A 56-year-old man complains of enlargedlower lip, pain induced by hot, sour, salty, andbitter foods, and lips gluing together in themorning. The lower lip has been graduallyenlarging and developing lumps his whole life.On examination: the lower lip is enlarged.The middle third of the Klein’s zone presentswith several small red dots with openings thatdischarge clear drops. The red border is dry andpeeling, there are fissures and erosions. What isthe most likely diagnosis?

A. Cheilitis glandularisB. Cheilitis exfoliativaC. Cheilitis actinicaD. Allergic contact cheilitisE. Atopic cheilitis

17. A 16-year-old young man complains oftemperature increase up to 38.7oC, pain wheneating and swallowing, foul acrid smell from hismouth. Lymph nodes, especially cervical ones,are enlarged, mobile, and painless. Objectivelythe patient presents with generalized hyperemiaof the oral mucosa, multiple petechiae, necroticspots, and profuse coating of the anteriorpharynx. Blood test: increased ESR, markedleukocytosis, monocytosis, atypical mononuclearcells, thrombocytopenia. What is the most likelydiagnosis?

A. Infectious mononucleosisB. Vincent stomatitis (acute necrotizing ulcerativegingivitis)C. Herpetic anginaD. Monoblastic leukemiaE. Acute herpetic stomatitis

18. A 57-year-old retired man complains ofattacks of burning pain and rashes on the skin ofhis face and oral mucosa on the right. Anamnesis:a course of radiation therapy for treatmentof gastric disease, past case of chickenpox.Objectively: along the third branch of thetrigeminal nerve the skin of the face presents with

Page 4: Krok 2 - testcentr.org.ua · D. Monoblastic leukemia E. Acute herpetic stomatitis 18. A 57-year-old retired man complains of attacks of burning pain and rashes on the skin of his

Терапевтична стоматологiя 4

isolated erosions covered in fibrinous coating.There are multiple vesicles on the hyperemic andswollen oral mucosa. Right-sided lymphadenitisis observed. What is the most likely diagnosis?

A. Herpes zosterB. NeuralgiaC. MurrainD. Acute recurrent herpesE. Neuritis

19. A patient consulted a dentist about a cosmeticdefect in the cervical region of the upper andlower canines. Various stimuli cause no pain.Objectively: there are V-shaped defects on thevestibular surface in the cervical area of theupper and lower canines. Their surface is smooth,glossy, hard. There is no reaction to probingand cold stimuli. What treatment should beadministered?

A. Filling of the defectsB. Metal crownsC. Applications with 10% solution of calciumgluconateD. Application of fluorine lacquerE. Medical intervention is unnecessary

20. A 35-year-old woman has complaints ofcosmetic defects of the front upper teethcrowns. The defects have been aggravatingfor the last 10 years. The patient suffers fromunpleasant sensations when brushing her teeth,and when chemical stimuli are applied. Objectiveexamination revealed defects localized in theenamel of the front upper teeth vestibularsurface. The defects are oval, saucer-shaped, andhave clear margins. Response to probing and coldstimuli was positive. Make the diagnosis:

A. Enamel erosionB. Enamel hypoplasiaC. Cuneiform defectD. Chemical necrosis of the toothE. Hyperesthesia of tooth hard tissues

21. A patient complains of dull ache in the 16tooth, which occurs during eating cold food.Previously the tooth had been filled due to deepcaries, the filling was lost 1 year ago. Objectively:a deep carious cavity that does not communicatewith the tooth cavity is present; percussion ispainless, probing is painful along the wholefloor of the carious cavity. Electric pulp test - 50microamperes. Thermodiagnosis is painful. Makethe diagnosis:

A. Chronic fibrous pulpitisB. Chronic deep cariesC. Chronic fibrous periodontitisD. Acute deep cariesE. Chronic gangrenous pulpitis

22. A 20-year-old man complains of sharp pain inthe mouth, increase of body temeperature upto 38, 5oC, headache and aching joints, generalweakness. The disease onset was 3 days ago dueto overexposure to cold. Objectively: the redborder is covered with hemorrhagic scabs, oralmucosa has large erosions and ulcers mergingwith each other and covered with grayish-white coating against the background of diffusehyperemia. Conjunctivitis is observed. The skin

of the forearms has erythematous spots 1,5 cm indiameter, with blisters in their center. What is themost likely diagnosis?

A. Stevens-Johnson syndromeB. Erythema multiformeC. Drug-induced stomatitisD. Lyell’s syndromeE. Behcet’s syndrome

23. A 22-year-old woman complains of paincaused by hot food and bursting sensation inthe tooth. Half a year ago she presented withbrief bouts of pain in the night, which over timeincreased in duration. Objectively: there is a largecarious cavity in the 24 tooth, which opens to thedental cavity, deep probing is painful. Electricpulp test is 80 microamperes. What is the mostlikely diagnosis?

A. Chronic gangrenous pulpitisB. Chronic concrementous pulpitisC. Chronic hypertrophic pulpitisD. Acute suppurative pulpitisE. Chronic fibrous pulpitis

24. A 40-year-old man had his root canal of the 34tooth filled due to chronic fibrous periodontitis.Soon the treated place became painful. On X-raythe root canal of the 34 tooth is filled to the rootapex. What tactics should the dentist choose tomanage the pain?

A. To prescribe physiotherapeutic proceduresB. To rinse with antiseptic mouthwashC. To make insicion along the mucogingival foldD. To provide conduction anesthesiaE. To provide infiltration anesthesia

25. A 35-year-old patient has been diagnosedwith chronic median caries of the 36 tooth. Thereis a Black’s class II cavity affecting masticatorysurface. What material should be chosen for thetooth filling?

A. Light-cure microhybrid compositeB. Glass ionomer cementC. Silicophosphate cementD. Light-cure fluid compositeE. Light-cure microfilled composite

26. A 49-year-old woman complains of cosmeticdefect of the 11, 21, and 22 teeth, whichdeveloped over a year ago. Objectively: on thevestibular surface at the equator of the 11, 21, and22 teeth there are shallow cup-shaped enameldefects that are dense on probing. Cold waterinduces no pain. Make the provisional diagnosis:

A. Enamel erosionB. Cuneiform defectC. Superficial cariesD. HypoplasiaE. Fluorosis

27. A 47-year-old patient complains of a burningsensation and pain in the mouth. Objectively:on the mucous membrane of cheeks along theline of teeth contact and in the corners ofthe mouth there are multiple polygonal brightred erosions 1,0-1,5 cm in diameter locatedon the hyperkeratinized plaque and opaquewhitish mucosa. Cytological analysis revealedkeratinizing epithelial cells. What is the most

Page 5: Krok 2 - testcentr.org.ua · D. Monoblastic leukemia E. Acute herpetic stomatitis 18. A 57-year-old retired man complains of attacks of burning pain and rashes on the skin of his

Терапевтична стоматологiя 5

likely diagnosis?

A. Leukoplakia, erosive formB. Lichen ruber planus, erosive formC. Erythema multiformeD. Secondary syphilisE. Lupus erythematosus, erosive form

28. A 38-year-old woman complains of burningpain in her lips and angles of her mouth, theirdryness. Anamnesis states that she has beensuffering from diabetes mellitus for the last 8years. Objectively: the vermillion border is dry,congestively hyperemic, covered in scales ofvarying size. In the angles of the mouth thereare fissures covered in white coating, the skin ismacerated. What ointment should be prescribedfor topical treatment in the given case?

A. ClotrimazolB. InterferonC. PrednisoloneD. LanolinE. Erythromycin

29. A 35-year-old woman complains of lipsenlargement. The first incident occurred one yearago, when she developed lip edema that abatedquickly, but the lips remained slightly enlarged.Three days ago after overexposure to cold herlips enlarged again. Objectively: ptosis, upperand lower lips are markedly enlarged, more onthe left, soft, elastic, and painless on palpation;no impressions on the lip surface are left afterpressing it with a finger. The tongue is swollen,with tuberous surface and folds on its back. Whatis the most likely diagnosis?

A. Melkersson-Rosenthal syndromeB. Miescher’s granulomatous cheilitisC. Quincke’s edemaD. Achard’s syndromeE. Meige’s trophedema

30. A 45-year-old man complains of dryness andpain in the lower lip. On examination: the lowerlip is swollen, dry, covered in small scales andfissures. In the Klein area (wet-dry line) there aredilated openings of salivatory glands observed asred dots producing clear substance. The lowerlip mucosa is lumpy. What is the most likelydiagnosis?

A. Glandular cheilitisB. Actinic cheilitisC. Meteorological cheilitisD. Eczematous cheilitisE. Exfoliative cheilitis

31. A 23-year-old man complains of gum bleedingwhen he brushes his teeth or eats solid food.Objectively: the gums of the front lower jaw arehyperemic, swollen and bleeding when palpated.Oral and gingival mucosa in other areas are notaffected. The occlusion is deep. The teeth arefirm, except for the 41 and 31 (degree 1 mobility).X-ray shows resorption of the alveolar septum inthe area of the 41, 42, 32, and 31 teeth up to 1/3 ofthe root length. What is the most likely diagnosis?

A. Localized periodontitisB. Generalized periodontitis, initial stageC. Generalized periodontitis, stage ID. Catarrhal gingivitisE. Parodontosis, stage I

32. A 40-year-old man, a chemical industryworker, notes the sour sensation in his mouth,pain response to thermal and chemical stimuli.On examination: on the vestibular surface andcutting edge of the front teeth there are chalkyenamel defects with uneven scalloped margins.Make the diagnosis:

A. Acidic necrosis of enamelB. Superficial cariesC. Enamel hypoplasia (erosive form)D. Fluorosis (erosive form)E. Median caries

33. A 42-year-old woman complains of acute lipenlargement, itching, and bursting sensation. Sheascribes her condition to introduction of a newlipstick. On examination the lips are significantlyenlarged and turgid, on palpation they are firm,elastic, and painless. Regional lymph nodesare without changes. What is the most likelydiagnosis?

A. Allergic contact cheilitisB. Meteorological cheilitisC. Exfoliative cheilitisD. Glandular cheilitisE. Actinic cheilitis

34. A 50-year-old patient, an employee of theprint shop, complains of foul smell from hismouth and excessive salivation. Objectivelyagainst the background of hyperemic and slightlyswollen gums there is a blue-black border alongthe gum margin of the lower jaw and upper frontteeth. There is a large amount of dental depositobserved. Name the type of stomatitis in thispatient:

A. LeadB. MercuryC. BismuthD. CatarrhalE. Necrotizing ulcerative

35. A 25-year-old woman consulted a dentistabout acute pain in her upper jaw on the left. Thepain occurs during eating. Objectively: on thedistal approximal surface of the 26 tooth there is acavity filled with light soft dentin. Probing causesslight pain along the dentin-enamel junction,percussion is painless. Cold water causes quicklyabating pain. What is the most likely diagnosis?

A. Acute median cariesB. Chronic median cariesC. Acute deep cariesD. Chronic fibrous pulpitisE. Chronic deep caries

36. A 19-year-old young man complains ofcosmetic defect of all his teeth, which developedimmediately after the teeth eruption. Objectivelyon the vestibular and masticatory surfaces of allpatient’s teeth there are enamel defects, toothcrowns present with dark brown discoloration.Percussion and probing are painful. In this area

Page 6: Krok 2 - testcentr.org.ua · D. Monoblastic leukemia E. Acute herpetic stomatitis 18. A 57-year-old retired man complains of attacks of burning pain and rashes on the skin of his

Терапевтична стоматологiя 6

fluoride levels in water are 2.6 mg/L. Make theprovisional diagnosis:

A. FluorosisB. Systemic hypoplasiaC. Chronic initial cariesD. Enamel erosionE. Chronic superficial caries

37. A 27-year-old man complains of achinglong-lasting pain in the 15 tooth during eating,especially cold food. Sometimes the pain occurswhen the temperature changes. Objectively: onthe distal surface of the 15 tooth there is acavity filled with softened dentin. Probing ispainful. Electroexcitability of the pulp is 35microamperes. What is the most likely diagnosis?

A. Chronic fibrous pulpitisB. Acute deep cariesC. Chronic deep cariesD. Hyperemia of the pulpE. Exacerbation of chronic pulpitis

38. A 35-year-old man complains of soursensation in his mouth and front teeth sensitivityto thermal and mechanical stimuli. Objectiveexamination revealed visible changes in theenamel of 13, 12, 11, 21, 22, and 23. The enamel isdull, rough, missing on the cutting edge. Probingof the vestibular surface of these teeth is painful,response to thermal stimuli is positive. Thepatient’s medical record states his occupation inindustrial production of inorganic acids. What isthe most likely diagnosis?

A. Necrosis of dental hard tissuesB. FluorosisC. Enamel erosionD. Pathologic teeth grindingE. Enamel hypoplasia

39. A 53-year-old patient complains of an ulceron the lateral surface of the tongue. The ulcerappeared 6 months ago as the result of a traumacaused by sharp tip of the 37 tooth metal crown.A dentist replaced the crown with the one ofbetter quality and prescribed keratoplastic drugs.Despite these measures the ulcer continues togrow. Lately there has been pain observed duringtalking, chewing, swallowing; sometimes the painirradiates to the pharynx. Objectively on thelateral surface of the tongue there is a painfululcer with uneven raised dense margins andlumpy floor with grayish necrotic coating. Whatis the most likely diagnosis?

A. Cancer of the lateral surface of the tongueB. Trophic ulcerC. Traumatic ulcerD. Vincent’s necrotizing ulcerative stomatitisE. Tuberculous ulcer

40. A 35-year-old patient, a veterinarian, came toa dentist with complaints of chills, fatigue, feverup to 38oC, muscle pain, sensations of dryness,burning, and pain in the oral cavity, excessivesalivation, vesicles in the interdigital folds, on thelips, oral and nasal mucosa. On examination ofthe oral cavity there were detected painful brightred erosions with polycyclic contours against thebackground of inflammation. The following wasobserved: scabs on the lips, enlarged tongue,

impaired speech, salivation up to 4 liters per day.Make the diagnosis:

A. MurrainB. Acute herpetic stomatitisC. ChickenpoxD. MeaslesE. Erythema multiforme

41. A man complains of short-term pain attackscaused by cold stimuli in the tooth on his upperleft jaw, which have been observed for the last 3weeks. Objective examination of the 25 revealeda Black’s I class carious cavity located withinmantle dentin. The cavity has narrow opening, itswalls and floor are covered with softened dentin.Probing is painful along the dentin-enamelborder, percussion is painless, thermometry ispainful, the pain quickly abates after removal ofa stimulus. Electric pulp test is 6 microamperes.Make the diagnosis:

A. Acute median cariesB. Acute deep cariesC. Chronic median cariesD. Chronic fibrous pulpitisE. Chronic deep caries

42. A 54-year-old man complains of tooth hardtissue defects on the upper and lower jaws.Objectively: in the precervical area of the upperand lower premolars within the external layer ofdentin there are hard tissue defects with surfacesmeeting under the angle. Surface of the defects issmooth, glossy, and dense. What measure shouldbe taken to prevent further progression of themorbid process in the patient?

A. To train him in efficient toothbrushingB. To perform functional examination of thethyroidC. To prescribe calcium preparations intakeD. To limit sour foods in the dietE. To limit sweet foods in the diet

43. A 28-year-old patient complains of pain andbleeding of gums in the frontal part of the upperjaw on the left. Two years ago, the 22 toothwas covered with a porcelain-fused-to-metalcrown. Objectively: interdental papilla betweenthe 21 and 22 tooth is hypertrophied, markedlyhyperemic, overlaps the crown of the 22 by 1/3of its height, bleeds when touched. Periodontalpocket between the 21 and 22 tooth is 4 mmdeep. Artificial crown is located on the gingivalmargin. Radiography reveals resorption of theinteralveolar septa between the 21 and 22 toothby 1/3 of their height. Specify a priority action inthe treatment of this patient:

A. Removal of the artificial crownB. GingivectomyC. Anti-inflammatory therapyD. Curettage of the periodontal pocketE. Sclerotherapy

44. A woman complains of pain in her gums,unpleasant smell from her mouth, difficult eating,general weakness, low-grade fever. Objectivelyher gums are hyperemic, with areas of ulceration,covered in necrotic deposit. Microscopy revealedfusospirochetosis. Choose the medication foretiotropic treatment:

Page 7: Krok 2 - testcentr.org.ua · D. Monoblastic leukemia E. Acute herpetic stomatitis 18. A 57-year-old retired man complains of attacks of burning pain and rashes on the skin of his

Терапевтична стоматологiя 7

A. MetronidazoleB. KeratolineC. GalascorbinD. ChlorhexidineE. Chymotrypsin

45. A man complains of gingival bleedingthat has been persisting for the last 2 years.Objectively he presents with chronic diffusecatarrhal gingivitis, teeth mobility is of the Idegree, periodontal pockets are 2-3 mm deepwith small amount of serous exudate, occlusionis markedly traumatic. X-ray shows damagedcortical plate, enlarged periodontal fissure inthe apical areas of the interalveolar septa,osteoporosis, and interalveolar septa resorptionby 1/3 of their height. Make the diagnosis:

A. Generalized periodontitis, stage I, chronicdevelopmentB. Chronic catarrhal gingivitisC. Parodontosis, stage ID. Generalized periodontitis, stage I, exacerbateddevelopmentE. Generalized periodontitis, early stage, chronicdevelopment

46. An 18-year-old patient complains of a whitespot on the vestibular surface of the 21 tooth.Objectively: the white spot is located near thecutting edge. The spot surface is glossy, its sizeremains unaltered on drying. Make the diagnosis:

A. Local hypoplasiaB. FluorosisC. Initial cariesD. Enamel necrosisE. Amelogenesis imperfecta

47. A 23-year-old man complains of acute gingivalbleeding and unpleasant smell from the mouththat appeared 5 days ago. Objectively gingivalpapillae and marginal gingiva are friable, brightred, swollen, painful, and bleed profusely onpalpation. Gingival pockets are 3 mm deep. X-rayshows marked osteoporosis of the interalveolarsepta, periodontal fissure in the apical areas ofthe interalveolar septa is enlarged. Cortical plateis intact. Make the diagnosis:

A. Acute catarrhal gingivitisB. Acute leukemiaC. Acute necrotizing ulcerative gingivitisD. Generalized periodontitis, stage II, exacerbateddevelopmentE. Hypovitaminosis C

48. A 20-year-old man complains of spontaneouspain in the 24 tooth, which arose suddenlyand persists for about 15 minutes. Objectively:the distal surface of the 24 tooth exhibits adeep carious cavity with overhanging walls.The cavity is filled with light softened dentinand communicates with the tooth cavity. Thecold stimulus causes acute, slowly abating pain.Percussion causes no pain response. Select thebest method of treatment:

A. Vital extirpationB. Vital amputationC. Biological methodD. Devital amputationE. Devital extirpation

49. A 25-year-old patient complains of pain whenbiting on the 15 tooth. The pain arose two daysago, has a constant aching nature and increasedsignificantly over the last day. Objectively:the crown of the 15 tooth is gray, the medialcontact surface exhibits a deep carious cavitycommunicating with the tooth cavity. Percussioncauses acute pain, the gingival mucosa in theprojection of the 25 tooth root apex is hyperemic.The regional lymph node is tender. Radiographshows an ill-defined zone of periapical bonedestruction. What is the most likely diagnosis?

A. Exacerbation of chronic periodontitisB. Acute serous periodontitisC. Acute suppurative periodontitisD. Chronic granulating periodontitisE. Acute serous periodontitis, intoxication stage

50. A 27-year-old patient has been referred bya prosthodontist for endodontic treatment ofthe 45 tooth. Objectively: the 45 tooth crownis destroyed; the lateral surface of the tongueand the buccal mucosa have patches of grayishmacerated epithelium slightly protruding abovethe mucosa surface at the points of direct contactwith the 45 tooth. The uvula and palatal bars arestagnant-red in colour; hard palate has papulaesurrounded with red margin and covered ingrayish epithelium. The submandibular, cervical,supraclavicular, and subclavicular lymph nodesare enlarged and painless. What is the provisionaldiagnosis?

A. Secondary syphilisB. Chronic recurrent aphthous stomatitisC. Lupus erythematosus, patch stageD. Soft leukoplakia (leucoplakia mollis)E. Lichen ruber planus

51. A patient complains of fever up to 38oC,headache, pain in the joints anf muscles, vesiclesin the oral cavity, mainly in the frontal part.Eating is sharply painful. For the last severalyears the disease has been recurring duringwet and windy weather. The patient oftensuffers from cases of URTI. Objectively: on thebuccal, lingual, labial mucosa there are confluenterosions against the erythematous background,with gray-white coating. There are bloody scabson the vermillion border and in the angles of themouth. Make the diagnosis:

A. Erythema multiforme exudativumB. SyphilisC. Acute aphthous stomatitisD. Acute necrotizing ulcerative stomatitisE. Chronic recurrent aphthous stomatitis

52. A woman came to a dentist for consultation.She is 4 months pregnant. Objectively: marginalgingiva presents with dense gingival papillae,torus-shaped, oval, enlarged up to 1/3 of the toothcrowns. Make the provisional diagnosis:

Page 8: Krok 2 - testcentr.org.ua · D. Monoblastic leukemia E. Acute herpetic stomatitis 18. A 57-year-old retired man complains of attacks of burning pain and rashes on the skin of his

Терапевтична стоматологiя 8

A. Hypertrophic gingivitisB. Catarrhal gingivitisC. PeriodontitisD. PeriodontosisE. Ulcerative gingivitis

53. A 53-year-old man complains of increasedteeth sensitivity to chemical stimuli. Objectively:the gums are pale pink, roots are bared by 1/3of their length. Small amount of dental depositis observed. The 15, 14, and 24 present withcuneiform defects. Probing of the bared cervicesand defects is painful. What is the most likelydiagnosis?

A. Periodontosis, I degreeB. Catarrhal gingivitisC. Periodontitis, II degreeD. Periodontitis, I degreeE. Ulcerative gingivitis

54. A 24-year-old man complains of painfuland bleeding gums. The condition onset was 3days ago after the patient had a case of acuterespiratory disease. Objectively the gingivalmucosa is swollen, bright-red, bleeds on probing,painful on palpation; tips of the gingival papillaare rounded; soft dental deposit is observed. X-ray shows no changes in the bone. What is themost likely diagnosis?

A. Catarrhal gingivitisB. Hypertrophic gingivitisC. Atrophic gingivitisD. PeriodontitisE. Ulcerative gingivitis

55. A 48-year-old man complains of gingivalovergrowth (”gums cover the teeth”). The patientsuffers from epilepsy and takes anticonvulsantagents. Objectively gingival papillae are ofnormal color, dense, with lumpy surface; they donot bleed on probing and cover the lower frontteeth up to their incisal surfaces. What is the mostlikely diagnosis?

A. Hypertrophic gingivitis, fibrous form, degreeIIIB. Hypertrophic gingivitis, edematous form,degree IIIC. Gingival fibromatosisD. Hypertrophic gingivitis, fibrous form, degree IIE. Hypertrophic gingivitis, edematous form,degree II

56. A 21-year-old man came to the dentistcomplaining of general weakness, muscle pain,body temperature up to 38.3oC, indigestion,excessive salivation, and rashes in the oral andnasal cavities, urethra, on the wings of the nose,and in the interdigital folds. These symptomsappeared after ingestion of milk during thepatient’s stay in the village. What is the mostlikely diagnosis?

A. MurrainB. Herpetic stomatitisC. Herpes zosterD. Behcet’s diseaseE. Infectious mononucleosis

57. A 28-year-old man complains of painless sorein his mouth that persists despite the attempts atself-treatment. Objectively the regional lymph

nodes on the left are enlarged and painless.Mucosa of the left cheek presents with roundulcer, 1 cm in diameter, with raised margins andcartilage-like infiltration in its basis. The surfaceof the ulcer is colored meat red and painless onpalpation. What is the most likely diagnosis?

A. Primary syphilisB. CancerC. Secondary syphilisD. Lupus vulgarisE. Decubitus ulcer

58. A 22-year-old woman came to a dentist forpreventive examination. During examination ofthe oral cavity the dentist detected a defect ofhard tooth tissues in the cervical area of the 22tooth within mantle dentin. The dentin is denseand pigmented. No reaction to percussion andprobing is observed. Make the diagnosis:

A. Chronic median cariesB. Cuneiform defectC. Acute deep cariesD. Acute median cariesE. Necrosis of hard tooth tissues

59. On examination of a 27-year-old patient thetip of the dental probe caught on the fissuresof the 36, 37, and 38 teeth. Margins of theenamel defect are dark, the surface is coarse.Teeth transillumination with photopolymer lamprevealed the defect to be limited to the enamel.What is the most likely diagnosis?

A. Chronic superficial cariesB. Chronic median cariesC. Acute superficial cariesD. Chronic initial cariesE. Acute initial caries

60. Carious cavities of the 11 and 21 teeth weredetected during the preventive examination ofa 20-year-old patient. What material should beused to fill the detected cavities?

A. Microhybrid compositeB. Macrofilled compositeC. AmalgamD. Phosphate cementE. Plastic

61. A patient complains of periodical gingivalhemorrhages during tooth brushing andincreased teeth sensitivity to thermal andchemical stimuli, which persist for the last 6years. On examination the gums are swollen andhyperemic. Periodontal pockets are 5 mm deepwith serous purulent content, tooth cervices arebared, I degree tooth mobility is observed. OnX-ray: irregular resorption of of alveolar septaup to their 1/2. What diagnosis corresponds withthe given clinical presentation?

A. Exacerbation of generalized periodontitis, IIdegreeB. Papillon-Lefevre syndromeC. Periodontosis, I degreeD. Exacerbation of generalized periodontitis, IdegreeE. Exacerbation of severe catarrhal gingivitis

62. A 24-year-old woman came to a dentist toreceive sanation. Objectively on the masticatory

Page 9: Krok 2 - testcentr.org.ua · D. Monoblastic leukemia E. Acute herpetic stomatitis 18. A 57-year-old retired man complains of attacks of burning pain and rashes on the skin of his

Терапевтична стоматологiя 9

surface of the 37 tooth there is a deep cariouscavity connected with the dental cavity. Thecavity probing is painless, no reaction to thermalstimuli is observed in the tooth, percussion ispainless. EOD is 108 microamperes. X-ray showstraces of filling material in the rooth canal ofthe 37 tooth, periodontal fissure is enlarged anddeformed. Make the diagnosis:

A. Chronic fibrous periodontitis of the 37 toothB. Chronic granulating periodontitis of the 37toothC. Chronic granulomatous periodontitis of the 37toothD. Chronic fibrous pulpitis of the 37 toothE. Exacerbation of chronic granulomatousperiodontitis of the 37 tooth

63. A 25-year-old man complains of short-termpain in the tooth on the lower right jaw duringeating sweet, hot, and cold food. Objectively: inthe 36 tooth on the distal surface there is a cariouscavity non-communicating with the dental cavity,dentin is softened. Probing of the cavity floor ispainful, percussion is painless. Electric pulp testis 16 microamperes. Make the final diagnosis:

A. Acute deep cariesB. Acute median cariesC. Pulpal hyperemiaD. Chronic gangrenous pulpitisE. Chronic fibrous periodontitis

64. A 30-year-old woman came to the dentistwith complaints of uncomfortable sensationof pressure in her upper right tooth, whichaggravates in response to hot stimulus, andfoul smell from the mouth. Objectively: thereis a deep carious cavity in the 17 tooth, whichcommunicates with the tooth cavity. Deepprobing causes severe pain, percussion of the 17tooth is painful. X-ray: there is slight wideningof the periodontal fissure near the root apex.Electric pulp test - 70 microamperes. What finaldiagnosis can be made?

A. Chronic gangrenous pulpitisB. Chronic fibrous pulpitisC. Acute purulent pulpitisD. Chronic fibrous periodontitisE. Exacerbation of chronic fibrous periodontitis

65. Medical committee registers the patients,who for a long time lived in an area pollutedwith radiation. The patients are advised onthe diet that will quickly purge the body fromradionuclides. The portion of products rich inpectine should be increased in their diet. Namethese products:

A. Fruits and vegetablesB. Meat productsC. PastaD. Dairy productsE. Baked goods

66. During carious cavity preparation in a20-year-old man, the pulp-chamber floor wasaccidentally perforated and horn of the pulpwas exposed. On the carious cavity floor thereis a point-like puncture surrounded with whitepredentin. Pink pulp can be seen through theperforation, pulp probing is acutely painful. What

treatment should be given to the patient?

A. Biological approachB. Vital amputationC. Vital extirpationD. Devital amputationE. Devital extirpation

67. A 37-year-old woman came to the dentist withcomplaints of brief attacks of toothache causedby eating sweets. Objectively there is a shallowcarious cavity within enamel. On probing cavitywalls and bottom are coarse; there is no responseto thermal stimuli. Make the diagnosis:

A. Acute superficial cariesB. Endemic fluorosisC. Enamel hypoplasiaD. Acute median cariesE. Chronic median caries

68. A 24-year-old patient came to the dentistcomplaining of chalky lesions on the front teeth.Objectively teeth 13, 12, 11, 21, 22, and 23present with chalky lesions separated by areasof healthy unchanged enamel. Lesion surface iscoarse; there is no response to thermal stimuli.Childhood years of the patient were spent in thearea with fluorine level of 1.8 mg/L in drinkingwater. Make the diagnosis:

A. Endemic fluorosisB. Enamel hypoplasiaC. Enamel hyperplasiaD. Acute superficial cariesE. Chronic superficial caries

69. A 48-year-old patient has addressed a hospitalwith complaints of defects in the paragingivalarea and slight sensitivity to thermal stimuli.Objectively there are hard tissue defects thatresemble a wedge with smooth polished walls onthe precervical vestibular surface of the 23 and24 teeth. Thermal test is slightly positive. What isthe most likely diagnosis?

A. Cuneiform defectB. Enamel necrosisC. Acute deep cariesD. Enamel erosionE. Endemic fluorosis

70. A woman complains of spontaneousattacks of acute pain, with practically nointermissions and irradiation into the temple;cold water slightly mitigates the pain. In tooth 26examiantion revealed deep carious cavity non-communicating with the dental cavity. Probingof the cavity bottom is acutely painful, thetooth is tender on vertical percussion. Make theprovisional diagnosis regarding tooth 26:

A. Acute suppurative pulpitisB. Acute diffuse pulpitisC. Pulpal hyperemiaD. Chronic fibrous pulpitisE. Chronic hypertrophic pulpitis

71. A 78-year-old man complains of a painful sorein his mouth that has been persisting for 2 monthsalready. The patient is a smoker. Objectively onthe buccal mucosa on the right there is a shallowulcer up to 1.5 cm in size with lumpy floor anduneven margins. There are yellowish granules on

Page 10: Krok 2 - testcentr.org.ua · D. Monoblastic leukemia E. Acute herpetic stomatitis 18. A 57-year-old retired man complains of attacks of burning pain and rashes on the skin of his

Терапевтична стоматологiя 10

its periphery. Palpation is painful, the lesion issoft. Regional lymph nodes are enlarged, painful,and matted together. What is the provisionaldiagnosis?

A. Tuberculous ulcerB. Cancerous ulcerC. Decubitus ulcerD. Trophic ulcerE. Hard chancre

72. A 63-year-old man complains of fever andmultiple painful rashes in his oral cavity and onhis face and torso. 3-4 days before the rashesappeared he had noticed a burning sensationfollowed by sharp shooting pain resemblingthat which occurs during lumbago. The patienttakes cytotoxic drugs for leukemia treatment.On examination there are multiple aphthaelocated in a row on the vermillion border andlabial, lingual, and buccal mucosa on the right.The aphthae have hyperemic borders, they arenot fused together and are sharply painful onpalpation. The right side of the face presents witherythematous spots, vesicles, and erosions. Makethe provisional diagnosis:

A. Herpes zosterB. Toxic allergic dermatostomatitisC. Erythema multiforme exudativumD. Secondary syphilisE. Chronic recurrent herpes

73. A 34-year-old man presents with persistingdull pain in his tooth, which aggravates on biting.One week ago the tooth was treated for deepcaries. Objectively on the masticatory surface oftooth 36 there is a filling, percussion is painful,there is a supracontact observed in the area of36. X-ray shows unchanged periodontium. Whatmistake was made when tooth 36 was filled?

A. High fillingB. Filling without insulation layerC. Gingival attachment is disturbedD. Insulation layer exceeds borders of thedentinoenamel junctionE. Filling without medicinal substance sealedinside

74. A 35-year-old man complains of persistingpain in tooth 24, which intensifies on biting.Objectively on the distal masticatory surfaceof tooth 24 there is a deep carious cavity filledwith food debris. Percussion of cavity bottom ispainless, there is no pain response to thermalstimuli. Percussion of tooth 24 is acutely painful.X-ray shows no pathologic changes of periapicaltissues in the area of root apices of 24. What isthe most likely diagnosis?

A. Acute serous periodontitisB. Acute suppurative periodontitisC. Acute suppurative pulpitisD. Acute diffuse pulpitisE. Exacerbation of chronic periodontitis

75. A 28-year-old woman is diagnosed withchronic generalized periodontitis, II degree.The doctor prescribed her a mouthwash withchlorhexidine gluconate as a part of complextherapy. This drug belongs to the following groupof antiseptics:

A. DetergentsB. DyesC. HalogensD. OxidantsE. Acids and alkalis

76. A 22-year-old man complains of tearing,throbbing, constant, intensifying pain in the toothon the upper left jaw. The pain has been persistingfor 4 days. Objectively tooth 26 has deep cariouscavity non-communicating with the dental cavity.Probing is painless. Percussion is acutely painful.The tooth is mobile. Mucogingival fold in thearea of tooth 26 is painful on palpation. Make thediagnosis:

A. Acute suppurative periodontitisB. Acute serous periodontitisC. Acute suppurative pulpitisD. Exacerbation of chronic periodontitisE. Acute local pulpitis

77. A 33-year-old man, a metalworker, complainsof pain and itching in the gums, gingivalhemorrhages intensifying during tooth brushing.The onset of the disease was 1 year ago.Objectively: the gums in the area of upperand lower frontal teeth are hyperemic, swollen,and cyanotic. There are significant mineralizeddeposits on the teeth; the periodontal sockets are3 mm deep and produce small amount of serousdischarge. What is the most likely diagnosis?

A. Chronic generalized periodontitis, I classB. Chronic localized periodontitis, I classC. Exacerbation of chronic generalizedperiodontitis, II classD. Acute localized periodontitis, II classE. Generalized periodontosis, I class

78. A 28-year-old man presents with profusecaseous coating on the posterior third of theback of his tongue, soft palate, tonsils, andposterior wall of the pharynx. Submandibular,submental and deep cervical lymph nodes havebeen enlarged for 4 months. Two weeks agothe patient developed intermittent fever andgeneral fatigue. Select the correct sequence ofHIV diagnosing:

A. Enzymoimmunoassay, immunoblotting(Western-Blot)B. CD4 cell count, enzymoimmunoassayC. Complete blood count, viral loadD. Complete blood count, enzymoimmunoassayE. Viral cultivation, enzymoimmunoassay

79. A 38-year-old man complains of sensation ofa foreign body on his tongue and developmentof gag reflex during talking. The signs appearedafter the prolonged taking of antibiotics.Objective examination detected thickened andpigmented filiform papillae enlarged to 2-3 cmin size. Histological analysis detected papillarhyperplasia and marked keratinization withoutalteration of the surrounding tissues. What is themost likely diagnosis?

A. Black hairy tongueB. Median rhomboid glossitisC. Fissured tongueD. Glossitis areata exfoliativaE. Geographic tongue

Page 11: Krok 2 - testcentr.org.ua · D. Monoblastic leukemia E. Acute herpetic stomatitis 18. A 57-year-old retired man complains of attacks of burning pain and rashes on the skin of his

Хiрургiчна стоматологiя 11

1. A 60-year-old man complains of stabbi-ng pain near the root of the tongue on the ri-ght, which develops during eating, especiallysour food. Objectively: there is a swelling inthe right submandibular area. On palpation thesubmandibular gland is dense and enlarged.Excretory duct orifice of the right submandibulargland is dilated and produces mucopurulentsecretion. What is the most likely diagnosis?

A. Sialolithiasis of the submandibular glandB. Calculous sialadenitis of the sublingual glandC. Acute suppurative lymphadenitisD. Acute sialodochitisE. Adenophlegmon of the right submandibulararea

2. A 67-year-old patient complains of recurrenterosion on the red border of the lower lip.Objectively: the erosion is oval in shape, 0,8х1,3sm in size, covered in thin scabs that reveal glossysurface with punctate bleeding, when removed.There are atrophic areas of the red borderdetected. Infiltration elements are absent. Thesubmandibular lymph nodes are not enlarged.What is the provisional diagnosis?

A. Manganotti’s abrasive precancerous cheilitisB. Leukoplakia, erosive ulcerative formC. KeratoacanthosisD. Bowen’s diseaseE. Cheilitis glandularis

3. A 22-year-old patient complains of a pai-nful swelling in the right parotid gland. A weekearlier the patient received a cheek abrasionthat healed under the purulent crust. Over thepast two days the patient had been observingprogressing pain and fever up to 38, 6oC. Objecti-vely: there is a soft tissue edema in the rightparotid region, the skin is slightly strained, wi-thout discoloration. There is a dense painful infi-ltration 2,5x3,5 cm large, the skin over it exhibitslimited mobility. The mouth can be fully opened,the mucous membrane around the orifice of thesalivary duct is unchanged, saliva is transparent.What is the most likely diagnosis?

A. Acute lymphadenitisB. Exacerbation of chronic parotitisC. Abscess of the parotid-masseteric regionD. Acute non-epidemic parotitisE. Epidemic parotitis

4. A 33-year-old woman has been admitted to thedentofacial department with complaints of painand edema in the right submandibular region,body temperature rise up to 39, 5oC. Objectively:the patient has asymmetric face because of softtissue edema of the right submandibular region,palpation reveals a dense infiltration, the skinover it is hyperemic, cannot make a fold. The 46tooth has a deep carious cavity. What is the mostlikely diagnosis?

A. Submandibular phlegmon on the rightB. Acute submandibular sialadenitisC. Acute suppurative periostitis of the mandibleD. Acute suppurative submandibularlymphadenitisE. Acute right-sided osteomyelitis of the mandi-ble

5. A 56-year-old man complains of swelling andpain in his right parotid area. The swelling wasnoticed 5-6 months ago. Objectively right-sidedparesis of the facial muscles can be determi-ned. Palpation reveals there a modrately pai-nful tuberous tumor fused with surrounding ti-ssues. In the center of the tumor there is an areaof softening. Submandibular and cervical lymphnodes on the right are enlarged and dense. Themouth can be opened without restriction. Thereis no saliva outflow from the opening of the rightparotid gland. What provisional diagnosis can bemade?

A. Adenocarcinoma of the right parotid glandB. Chronic non-epidemic parotitisC. Chronic lymphadenitis of the right parotid areaD. Mixed tumor of the right parotid glandE. Actinomycosis of the right parotid gland

6. A 44-year-old patient consulted a dentalsurgeon about constant acute pain in the upperjaw region on the left that aggravates duringteeth joining. The pain appeared 3 days ago.Objectively: the face is symmetric, mouth openi-ng is not limited. The crown of the 26 tooth ishalf-decayed. Probing of the carious cavity ispainless. Percussion of the 26 tooth provokesacute pain. Mucous membrane of the alveolarprocess is edematic, hyperemic at the level of the26 tooth. The 26 tooth had been treated before.What is your provisional diagnosis?

A. Exacerbation of chronic periodontitis of the26 toothB. Acute suppurative periodontitis of the 26 toothC. Acute pulpitis of the 26 toothD. Acute suppurative periostitis of the left upperjaw extending from the 26 toothE. Periodontitis of the 26, 27, and 28 teeth

7. A man was diagnosed with hard palate abscess.What approach should be chosen for abscess di-ssection?

A. Triangular dissection of the hard palate areaB. Linear dissection parallel to the hard palaterapheC. Linear dissection perpendicular to the hardpalate rapheD. Pus aspiration with a syringeE. Abscess puncture

8. A 42-year-old woman complains of acute painin her lower jaw, teeth mobility, high fever. Thecondition persists for 2 days. On clinical exami-nation a doctor diagnosed her with acute mandi-bular osteomyelitis. What tactics regarding themobile teeth should the doctor choose?

A. Extraction of the causative tooth onlyB. Extraction of all mobile teethC. Conservative treatment of the causative toothD. Conservative treatment of all mobile teethE. Extraction of the causative tooth, trepanationand treatment of all mobile teeth

9. A 35-year-old man complains of thickening ofhis maxillary alveolar process. Preliminary di-agnosis of maxillary radicular cyst was made.What substance will be obtained as the result ofthe puncture of the alveolar process in the areaof buccal thickening?

Page 12: Krok 2 - testcentr.org.ua · D. Monoblastic leukemia E. Acute herpetic stomatitis 18. A 57-year-old retired man complains of attacks of burning pain and rashes on the skin of his

Хiрургiчна стоматологiя 12

A. Yellowish liquidB. BloodC. PusD. EpitheliumE. Turbid infiltration

10. Six months ago a 40-year-old man had histooth 26 extracted; afterwards his oral cavityand maxillary sinus became communicating andthe patient developed the first signs of maxi-llary sinusitis. What surgical procedure shouldbe performed in this case?

A. Maxillary sinusotomy with simultaneousplastic surgery for repair of the fistulaB. Caldwell-Luc surgeryC. Fistula plicationD. Fistula packing with Iodoform gauzeE. Osteotomy of the alveolar process

11. A 46-year-old woman complains of bleedi-ng gums, suppuration, teeth mobility. She hasbeen presenting with these signs for 10 years.On examination her upper and lower gums arehyperemic, swollen, bleed on touch. In the areaof 42, 41, 31, and 32 periodontal pockets are upto 8 mm deep, contain purulent discharge; theseteeth demonstrate mobility of the II degree,other teeth present with mobility of the I degree.In the area of 42, 41, 31, and 32 X-ray showsinteralveolar septa resorption by 1/2 of the rootlength and signs of osteoporosis. What is themost advisable method of surgical treatment inthis case?

A. OsteoplastyB. CurettageC. GingivotomyD. GingivectomyE. Flap surgery

12. A 49-year-old man was diagnosed withrecurrence of lower lip cancer two years afterhe had undergone radiation therapy. Objecti-vely in the area of his lower right lip there isa neoplasm 1x2 cm in size with an ulcer in itscenter. In the right submandibular area there are2 round, enlarged, dense, painless lymph nodes.What approach to the treatment would be opti-mal in this case?

A. Combined treatmentB. Wedge resection of the lower lipC. Rectangular resection of the lower lipD. Trapezial resection of the lower lipE. Vanakh’s operation

13. A 49-year-old man complains of progressi-ng reduction of mouth opening, pain on theleft when swallowing, severe deterioration ofhis general well-being, temperature increase upto 39.3oC. Destroyed tooth 38 presents withacute pain. Objectively the face is symmetri-cal, the submandibular lymph nodes on the leftare enlarged and painful on palpation. Palpationunder the left mandibular angle and in the theleft retromandibular area provokes sharp pain.Mouth opening and movement of the mandi-ble to the left are significantly reduced. The leftpterygomandibular fold is hyperemic and infi-ltrated. What is the most likely diagnosis?

A. Phlegmon of the pterygomandibular spaceB. Phlegmon of the parapharyngeal spaceC. Phlegmon of the retromandibular areaD. Phlegmon of the submandibular spaceE. Abscess of the sublingual fossa

14. The 15 tooth must be extracted. The toothcrown is retained. What instrument should beused in this case?

A. Forceps with S-shaped handlesB. Straight forcepsC. Bayonet forcepsD. Left-sided forceps with S-shaped handlesE. Right-sided forceps with S-shaped handles

15. A 32-year-old woman complains of tumor-like growth in the mucosa of her left cheek.Locally: buccal mucosa is of normal color. Inthe distal area there is a rounded elongatedgrowth, soft and elastic, attached to a pediclesized 0,5х1,5 cm. Make the provisional diagnosis:

A. PapillomaB. LipomaC. HemangiomaD. Pleomorphic adenomaE. Fibroma

16. A 52-year-old patient complains of pain andswelling in the right parotid region. These mani-festations have been present for about 2 years.Over the last month the swelling has enlarged,pain has intensified. Objectively: the face isasymmetric due to the dense infiltrate in the ri-ght parotid region. The poorly circumscribed,painful formation infiltrates the surrounding ti-ssues. At the right side of neck in front andbehind the sternocleidomastoid muscle there areenlarged, dense, mobile lymph nodes. The rightnaso-buccal groove is flattened, the corner of themouth is downturned. The mouth opens freely.The are pronounced symptoms of the right facialnerve paresis. What disease can be suspected?

A. Adenocarcinoma of the parotid salivary glandB. Chronic parotitisC. Actinomycosis of the parotid-masseteric regionD. Chronic lymphadenitisE. Pleomorphic adenoma of the parotid gland

17. A 57-year-old woman came to a dentist forextraction of the 34 tooth due to exacerbationof chronic periodontitis. What instrument wouldbe optimal for tooth extraction in the given case?

A. Beak-shaped non-crushing forcepsB. Beak-shaped crushing forcepsC. Beak-shaped curved forcepsD. Straight elevatorE. Curved elevators

18. A 28-year-old man complains of pain in theinfraorbital and parotid region on the left. Onexamination: hemorrhage occurs in the lowereyelid and conjunctiva of the left eye, there aresigns of crepitation and step deformity of theeyesocket lower edge. The mouth opens by 1 cm.Make the diagnosis:

Page 13: Krok 2 - testcentr.org.ua · D. Monoblastic leukemia E. Acute herpetic stomatitis 18. A 57-year-old retired man complains of attacks of burning pain and rashes on the skin of his

Хiрургiчна стоматологiя 13

A. Zygomatic bone fractureB. Malar arch fractureC. Left articular process fractureD. Traumatic arthritis of the temporo- mandibularjointE. Hematoma of the infraorbital region

19. A 65-year-old woman complains of aneoplasm in the area of the nasolabial fold onthe left, which appeared one month ago. Objecti-vely: there is a gray neoplasm on the skin of thenasolabial fold on the left, markedly keratotic,3,0х0,5х0,3 cm in size. Neoplastic base is pai-nless, dense, and elastic. What is the most likelypathology that results in such clinical presentati-on?

A. Cutaneous horn of the left nasolabial foldB. Common wart of the left nasolabial foldC. Senile keratosis of the left nasolabial foldD. Keratoacanthoma of the left nasolabial foldE. Lupus

20. A 55-year-old patient consulted a dentistabout a rounded tumor-like formation about 1cm in diameter located within the red border ofhis lower lip. Objectively: the tumor-like formati-on protrudes about 5 mm above the red border,is dense and grayish-red. The surface of theformation is covered with thin scales that canhardly be removed. What is the most likely di-agnosis?

A. Verrucous precancer of the red borderB. Manganotti’s abrasive precancerous cheilitisC. Precancerous limited hyperkeratosis of the redborderD. Bowen’s diseaseE. Erythroplasia of Queyrat

21. An injured patient complains of reducedopening of the mouth, nose bleeding, skinnumbness in the infraorbital and lower eyelidregion. Objectively: there is face deformationdue to the depression of soft tissues in the leftcheekbone region, step deformity in the middlepart of the inferior margin of the left orbit and inthe area of the zygomatic alveolar crest. What isthe most likely diagnosis?

A. Zygomatic bone fracture with displacement ofthe bone fragmentsB. Fracture of the right zygomatic bone withoutdisplacement of the bone fragmentsC. Le Fort I fracture of maxillaD. Le Fort II fracture of maxillaE. Fracture of the malar arch

22. A 30-year-old woman came to a dentist withcomplaints of a slightly bleeding ”sore” on herlower lip, which is located at its median and bi-sects the lip into two even parts. On palpation thelip is swollen and slightly painful. What diagnosiscorresponds with the given clinical presentation?

A. Chronic labial fissureB. Tappeiner’s leukoplakiaC. Erosive-ulcerative leukoplakiaD. Lichen ruber planus, erosive-ulcerative formE. Meteorological cheilitis

23. A 44-year-old woman complains of the faceswelling in the right lower jaw area and teeth

mobility. Objectively: soft tissues are withoutchanges, the regional lymph nodes cannot bepalpated. The alveolar process and the bodyof the lower jaw near the 46, 47, and 48 teethare thickened, painless when palpated, andlumpy. The teeth in the thickened area are mobi-le. Puncture consists of brown fluid withoutcholesterol crystals. What is the provisional di-agnosis?

A. OsteoclastomaB. AdamantinomaC. OsteomaD. Follicular cystE. Odontoma

24. A 30-year-old patient needs to have his26 tooth extracted because of exacerbation ofchronic periodontitis. Objectively: the crown ofthe 26 tooth is decayed by 1/3. What forceps canbe used for this tooth extraction?

A. S-shaped forceps with a projecting tip on theleft beakB. S-shaped forceps with a projecting tip on theright beakC. Straight forcepsD. Straight elevatorE. S-shaped forceps without projecting tips

25. A 50-year-old man was diagnosed with si-alolithiasis with the salivary gland stone locateddeep within the salivary gland. Choose the opti-mal treatment tactics:

A. Submandibular gland excisionB. Radiation therapyC. SclerotherapyD. Removal of the sialolith while retaining theglandE. Conservative pharmacotherapy

26. The department of dentofacial surgery admi-tted a patient who needs repair of a post-traumatic nose wing defect up to 3,0 cm in di-ameter. The trauma occured six months ago.What kind of grafting is indicated in this clini-cal situation?

A. Grafting with chondrocutaneous flap of theauricleB. Grafting with local tissues of nasolabial orcheek regionsC. Grafting with pedicle flap of frontal and buccalregionsD. Grafting with tubed pedicle flap (Filatov’s flap)E. Free grafting with dermal flap

27. A 30-year-old patient is diagnosed with acutesuppurative odontogenic periostitis of the upperleft jaw originating from tooth 23. The crownof 23 on the left is destroyed with caries by 1/3.Teeth 22 and 24 are intact. Spot-film X-ray showswidening of the periodontal fissure of 23. Whattreatment would be the most advisable in thiscase?

Page 14: Krok 2 - testcentr.org.ua · D. Monoblastic leukemia E. Acute herpetic stomatitis 18. A 57-year-old retired man complains of attacks of burning pain and rashes on the skin of his

Хiрургiчна стоматологiя 14

A. Periosteotomy and pharmacotherapy followedby treatment of the causative toothB. -C. Extraction of the causative tooth, physi-otherapyD. Extraction of the causative tooth,pharmacotherapyE. Extraction of the causative tooth, peri-osteotomy

28. A 30-year-old patient complains of painand swelling in the area of the left parotid sali-vary gland, which occurred 7 days after he hadundergone abdominal cavity surgery. Objecti-vely: body temperature is 39oC, reduced mouthopening, dry mouth; when the gland is massaged,there is purulent exudate being secreted fromits duct. The patient can be diagnosed with thefollowing disease:

A. Acute non-epidemic parotitisB. Acute epidemic parotitisC. Phlegmon of submasseteric spaceD. Parenchymatous parotitisE. Phlegmon of parotid-masticatory region

29. A 45-year-old man presents with facialasymmetry due to a dense isolated infiltrationin his right buccal area; the skin over the infi-ltration is cyanotic, thinned out; in the center ofthe infiltration there is a fistula. In the oral cavitythe crown of 46 is destroyed by 2/3, along themucogingival fold the band connecting the toothwith the fistula can be palpated. Make the di-agnosis:

A. Migrating facial granulomaB. Chronic mandibular osteomyelitisC. Cheek furuncleD. Odontogenic lymphadenitisE. Actinomycosis

30. A 35-year-old man has been hospitalized intoa dentofacial unit with complaints of mobility ofthe 38, 37, and 36 teeth and a fistulous tract inthe socket of the extracted 35 tooth. The condi-tion has been persisting for 3 months. Inserti-on of a grooved probe into the fistulous tractpalpated a bared coarse bone fragment that easi-ly moved under pressure. X-ray of the lower jawdemonstrates a focus of bone tissue destruction,with a spot of dense bone tissue 0.5х0.3 cm insize. Make the diagnosis:

A. Chronic osteomyelitisB. Acute osteomyelitisC. Exacerbation of chronic osteomyelitisD. Chronic periostitisE. Actinomycosis

31. After extreme overexposure to cold a 42-year-old patient complains of headache in theleft frontal lobe and the left upper jaw. Objecti-vely: the face is symmetrical; left nasal meatusbreathing is obstructed, and serous-purulentdischarge is being produced; palpation of thesuborbital area and further along the mucogi-ngival fold in the 24 and 25 teeth projectionreveals slight pain. Percussion of these teethis painless. The 24 tooth is filled. The alveolarprocess mucosa has no visible alterations. X-rayimaging shows decreased pneumatization of theleft maxillary sinus. What is the provisional di-

agnosis?

A. Exacerbation of chronic odontogenic maxi-llary sinusitisB. Acute periodontitis of the 24C. Exacerbation of chronic periodontitis of the 24D. Acute rhinogenous maxillary sinusitisE. Acute albuminous periostitis of the left maxilla

32. A patient with complaints of toothache inthe left upper jaw has made an appointmentwith a dental clinic. He was diagnosed withchronic periodontitis of the 24 tooth. What kindof anesthesia is necessary for painless extractionof the 24 tooth?

A. Infraorbital and palatinal anesthesiaB. Tuberal and palatinal anesthesiaC. Infraorbital and incisor anesthesiaD. Tuberal and incisor anesthesiaE. Surface and tuberal anesthesia

33. A 35-year-old woman complains of toothacheand thickened body of the mandible. Objecti-vely: the lower left jaw is thickened, Dupuytren’ssymptom is observed on the vestibular surface ofthe mucogingival fold in the area of the 36 and 37teeth. X-ray of the lower left jaw demonstratesthe rounded focus of bone tissue destructionwith clear margins. The roots of the 36 and 37teeth are resorbed. Puncture yielded brown li-quid. What is the most likely diagnosis?

A. Giant cell tumor of the body of mandibleB. Ameloblastoma of the body of mandibleC. Fibrous dysplasia of boneD. Cancer of the body of mandibleE. Sarcoma of the body of mandible

34. X-ray of the patient shows a focus of bonedestruction 3x4 cm in size in the mandibularbody. The focus is structured as numerous smallcavities different in size and shape and separatedby septa. Tumor puncture yielded brown liquid.What is the most likely diagnosis?

A. Giant cell tumor of the mandibleB. Radicular cyst of the mandibleC. Carcinoma of the mandibleD. Soft odontoma of the mandibleE. Mandibular ameloblastoma

35. A patient complains of the alveolar processdeformation on the left upper jaw. Objectively:the crown of the 25 tooth is destroyed with cari-osity. X-ray image of the paranasal sinuses showsthe left one to have veil-like shading with cleardome-shaped margin. X-ray image of the crownof the 25 tooth shows absence of the periodontalfissure at the the palatal root apex. What is themost likely diagnosis?

A. Radicular cyst that invaded in the maxillarysinusB. Chronic rhinogenous maxillary sinusitisC. Chronic odontogenic maxillary sinusitisD. Maxillary sinus mucosal cystE. Maxillary cancer

36. A 25-year-old woman made an appointmentwith the dental surgeon for oral cavity sanation.Objectively the crown of tooth 37 is destroyed by2/3. Gingival mucosa around tooth 37 is without

Page 15: Krok 2 - testcentr.org.ua · D. Monoblastic leukemia E. Acute herpetic stomatitis 18. A 57-year-old retired man complains of attacks of burning pain and rashes on the skin of his

Хiрургiчна стоматологiя 15

changes. What anesthesia should the dentalsurgeon give to the patient for the procedureof tooth extraction?

A. Mandibular and buccal anesthesiaB. Intraoral infraorbital nerve blockC. Tuberal anesthesiaD. Mandibular anesthesiaE. Mental nerve block

37. A 29-year-old man came to an oral surgerydepartment to extract the 38 tooth. There arecomplaints of pain and reduced mouth openi-ng. Objectively: body temperature is 38oC,general condition is satisfactory, slight facialasymmetry is observed due to soft tissue swellingunder the gonial angle on the left. Inflammatorycontracture of the III degree is observed. Inthe oral cavity there are edema and hyperemiaof mucosa along the pterygomandibular foldon the left. The submandibular lymph nodesare enlarged and painful on palpation. Whatconduction anaesthesia should be applied pri-or to the extraction of the 38 tooth?

A. Berchet-Dubov anaesthesiaB. Mandibular, lingualC. Buccal, lingualD. InfiltrationE. Tuberal

38. A 52-year-old man was referred to an in-patient dentofacial department with complaintsof pain in the submandibular area, aggravati-ng during eating. Anamnesis states frequentexacerbations of inflammatory processes. Mainand additional investigations resulted in provisi-onal diagnosis of submandibular sialolithiasis. Aconcrement 1,5 cm in diameter is localized in thebody of the gland. What tactics should a dentalsurgeon choose?

A. Gland extirpationB. Saliva-producing dietC. Physiotherapeutic proceduresD. Puncture biopsyE. Extract the concrement from the gland andplace a blind suture

39. A 34-year-old man complains of soft ti-ssues edema in his lower left jaw and fistulaein the submandibular area. Teeth 36 and 37are destroyed. Alveolar mucosa is swollen andhyperemic at the level of 36 and 37. X-raydetected sequestra in the mandibular body onthe left. What treatment method should bechosen in this case?

A. Extraction of teeth 36 and 37 and mandibularsequestrectomyB. Extraction of teeth 36 and 37C. Mandibular sequestrectomyD. Puncture of the inflamed areaE. Antibacterial treatment

40. A 42-year-old man was delivered to thehospital in the severe condition: inert, bodytemperature is 39.1oC, there is acutely painfulinfiltration of the mouth floor and submandi-bular area on the right. The skin over the infi-ltration is turgid and cyanotic. Palpation detectscrepitus under the skin. What diagnosis can bemade in this case?

A. Ludwig’s angina (suppurative-necroticphlegmon of the mouth floor)B. Adenophlegmon of the mouth floorC. Malignant tumor of the mouth floorD. Actinomycosis of the mouth floorE. Odontogenic phlegmon of the mouth floor

41. A patient diagnosed with rheumatoid arthri-tis came to a dental surgeon complaining ofpainful mouth opening characterized by painon both sides of the jaw. The following ischaracteristic of rheumatoid arthritis of thetemporomandibular joint:

A. Pain is observed on both sides of thetemporomandibular jointB. Pain is observed on one side of thetemporomandibular jointC. Pain can be observed on one or both sides ofthe temporomandibular jointD. Clicking is observed in the joint on the rightE. Clicking is observed in the joint on the left

42. After a blow to the temporomandibular joi-nt the patient developed facial hematoma, thejoint is difficult to move, mandibular mobility isreduced. Attempts to open the mouth wide arepainful. What examination should be performedto make the diagnosis?

A. Bilateral X-ray of the temporomandibularjoint with mouth open and closedB. Limit the joint mobilityC. X-ray and consultation with the neurologistD. Rheoencephalography and consultation withthe neurologistE. Panoramic dental X-ray

43. A 38-year-old man after a domestic acci-dent complains of pain and mobility of hisupper teeth, problems with eating. Objectively:soft tissues edema. The 11 and 21 teeth are di-splaced towards the palate, mobile (II degree),painful on percussion. Mucosa surrounding theaffected teeth is hyperemic and swollen. X-raydemonstrates widened periodontal fissure of the11 and 21. Choose the treatment method:

A. Setting of the teeth and their fixation with aflat occlusal splintB. Extraction of the 11 and 21 teethC. Reimplantation of the 11 and 21 teethD. Immobilization or mouthguardE. Removal of tooth pulp in the 11 and 21 teeth

44. During or immediately after an injection,certain local complications can develop. What isNOT one of those complications?

A. Mucosal necrosisB. Dermal ischemiaC. DiplopiaD. Functional paralysis or paresis of facial musclesE. Damage to a blood vessel by the needle

45. What manipulation of those listed belowis NOT a part of typical procedure of toothextraction with forceps?

Page 16: Krok 2 - testcentr.org.ua · D. Monoblastic leukemia E. Acute herpetic stomatitis 18. A 57-year-old retired man complains of attacks of burning pain and rashes on the skin of his

Хiрургiчна стоматологiя 16

A. Applying tip of forceps jaw to the edge ofalveolar processB. Applying forceps jaw to the toothC. Pushing forceps jaw to the cementoenameljunctionD. Closure of forceps handlesE. Tooth dislocation and extraction from thesocket

46. After sustained trauma a man developednose bleeding, reduced mouth opening, sensati-on of paresthesia in the right infraorbital regi-on and lower eyelid. Objectively the face isasymmetric due to concave right temporal regi-on; step deformity symptom is observed in themiddle of the lower right eye socket and in thearea of zygomaticoalveolar crest. What is themost likely diagnosis?

A. Displaced fracture of the temporal boneB. Le Fort II maxillary fractureC. Nondisplaced fracture of the temporal boneD. Le Fort I maxillary fractureE. Zygomatic arch fracture

47. A 24-year-old woman made an appointmentwith the dental surgeon for extraction of tooth38. What anesthesia should be given to the pati-ent for the procedure of tooth extraction?

A. TorusalB. MandibularC. TuberalD. InfiltrationE. Plexus

48. A patient complains of pain and sensati-on of heaviness in the left side of his face andmucous discharge from the nose. On exami-nation: left cheek edema, destroyed 26 tooth.Tooth percussion is sharply painful. X-raydemonstrates shadowed left maxillary sinus.What disease corresponds with the given clini-cal presentation?

A. Acute odontogenic maxillary sinusitisB. Acute rhinogenic maxillary sinusitisC. Chronic odontogenic maxillary sinusitisD. Cyst of the maxillary sinusE. Acute ethmoiditis

49. During application of tuberal anesthesiathe patient developed rapidly increasing ti-ssue edema and reduced mouth opening. Whatresulted in such a condition?

A. Vascular traumaB. Muscle trauma during anesthesia applicationC. Nerve trunk traumaD. Intolerance to the anestheticE. Anaphylactic shock

50. During application of infraorbital anaesthesiathe patient developed a postinjection hematoma.What vessel had been damaged?

A. Infraorbital arteryB. Maxillary arteryC. Pterygoid venous plexusD. Temporal arteryE. Palatine artery

51. A patient came to a dental surgeon with

complaint of periodical pains in the 22 tooth. OnX-ray examination the patient was diagnosed wi-th granulomatous periodontitis developed due tofilling material penetrating the space behind theroot apex. What further treatment tactics shouldbe chosen?

A. Resection of the root apexB. Extraction of the 22 toothC. Prescription of anaestheticsD. Referral to an oncologistE. Recurrent endodontic treatment

52. A woman came to the dental surgeon wi-th complaints of teeth mobility. After objecti-ve examination and X-ray analysis she was di-agnosed with generalized periodontitis of stagesI and II. Which teeth of those affected by peri-odontitis should be extracted?

A. With degrees 2-3 of tooth mobilityB. Intact teethC. Teeth with painful percussionD. With degree 1 of tooth mobilityE. Carious teeth

53. After the inflammatory process in the parotidarea a woman developed frequent pain attacksresembling electric current in her face on the ri-ght. The attacks last for 15-20 minutes. The mostlikely diagnosis is:

A. Trigeminal neuralgiaB. Tympanic plexus neuralgiaC. Trigeminal neuritisD. Exacerbation of chronic maxillary sinusitisE. Exacerbation of chronic osteomyelitis

54. A 40-year-old man came to an admissionroom with an incised wound of the infraorbitalregion received 8 hours ago. On examination thewound underwent primary surgical treatment. Incase of an incised wound its edges:

A. Should not be excisedB. Should be closed with secondary suturesC. Should be closed with primary delayed suturesD. Should be processed with antibiotic solutionE. Should be cleaned and drained

55. The patient with shallow vestibule of mouthand edentulous mandible underwent a surgery:a mucoperiosteal flap was relocated from thealveolar ridge to the body of the mandibleand fixed with denture acting as a bandage.What surgical procedure was used for vestibulardeepening?

A. RumpelB. KazanjianC. ThierschD. TraunerE. Rhermann

56. A 19-year-old young man complains of a fi-stula on the neck anterior surface, which peri-odically reappears at the same place. Objecti-vely: at the neck midline between the hyoidbone and thyroid cartilage there is a fistula;the skin of the affected area is scarred, drawn-in, and macerated. In the surrounding tissues adense band extending from the fistula opening tohyoid bone can be palpated. A doctor has made

Page 17: Krok 2 - testcentr.org.ua · D. Monoblastic leukemia E. Acute herpetic stomatitis 18. A 57-year-old retired man complains of attacks of burning pain and rashes on the skin of his

Хiрургiчна стоматологiя 17

a provisional diagnosis of thyroglossal fistula.Specify the additional method of investigation:

A. Contrast radiographyB. ProbingC. Computer tomographyD. UltrasoundE. -

57. A 40-year-old patient requires surgical sanati-on of the oral cavity. Objectively: the 36 toothis completely destroyed. Mouth can be fullyopened. What anaesthesia would be optimal forextraction of the 36 tooth?

A. TorusalB. MandibularC. MentalD. InfiltrationE. Berchet-Dubov

58. A 44-year-old man came to extract destroyedtooth 24. Objectively his face is symmetrical, thecrown of 24 is destroyed by 2/3, percussion ispainless. Gingival mucosa surrounding the toothis unchanged. X-ray shows enlarged periodontalfissure in the area of the root apex. What is themost likely diagnosis?

A. Chronic fibrous periodontitis of 24B. Chronic granulating periodontitis of 24C. Chronic granulomatous periodontitis of 24D. Exacerbation of chronic periodontitis of 24E. Chronic gangrenous periodontitis of 24

59. The maxillofacial surgery unit received apatient with complaints of inability to close hismouth. This condition occurred when the pati-ent was biting an apple. Objectively there is afrightened expression on the patient’s face, themouth is open wide, the chin is displaced to theleft, salivation is observed. Palpation through theexternal acoustic meatus detected no movementsof the right articular head. What is the most li-kely diagnosis?

A. Right temporomandibular joint dislocationB. Fracture of the mandibular processC. Acute temporomandibular arthritisD. Temporomandibular joint pain dysfunctionsyndromeE. Bilateral temporomandibular joint dislocation

60. A 19-year-old girl addressed an oncologistwith complaints of slowly growing tumor-likemass on the tip of her tongue. The mass was firstnoticed 5-6 years ago. The patient requested nomedical help. Objectively: there is a pale pinkround growth with wide pedicle on the tongueapex; the growth is painless, elastic; there areno changes of mucosa surrounding the pedicle.Submandibular lymph nodes cannot be palpated.What kind of tumor is it?

A. PapillomaB. AtheromaC. FibromaD. LipomaE. Keratoma

61. A 32-year-old patient addressed a dentist wi-th complaints of inability to close his mouth.Objectively the mouth is half-open, the chin is

protruding forwards and is displaced to the left.Such condition occurred after the mouth wasopened wide. What is the most likely diagnosis?

A. Anterior right-sided mandibular dislocationB. Anterior left-sided mandibular dislocationC. Anterior bilateral mandibular dislocationD. Posterior right-sided mandibular dislocationE. Posterior left-sided mandibular dislocation

62. A 37-year-old patient has symmetrical face;the mucosa in the area of the 12 tooth root apexprojection is pale pink; palpation is painless; thetooth crown is destroyed by 1/3; percussion ispainless. X-ray: the root canal of the 12 tooth isfilled to the apex; granuloma 4 mm in diameter islocated near the root apex. Choose the methodof surgical treatment:

A. Granuloma removal with root apex resectionB. Root hemisectionC. Coronary radicular tooth separationD. Root amputationE. Tooth extraction

63. A 48-year-old man presents with verruciform,dense, gray-white growths on the buccal mucosa.The growths protrude above the neighbouringtissues and are surrounded by keratinized gray-white spots that cannot be scraped off. Make theprovisional diagnosis:

A. Verrucous leukoplakiaB. Erosive leukoplakiaC. Bowen’s diseaseD. Erythroplasia of QueyratE. Papillomatosis

64. During preventive examination a patient wasdiagnosed with precancerous hyperkeratosis ofthe lower lip vermillion border. What treatmentshould be prescribed?

A. Surgical removal of the focus within healthytissuesB. Surgical removal of the focus within healthytissues + close-focus roentgenotherapyC. No treatment is necessaryD. Surgical removal of the focus within healthytissues + chemotherapyE. Palliative treatment

65. A 23-year-old patient is hospitalized into adentofacial unit with provisional diagnosis of theII degree thermal burns of the right buccal andparotid-masseter region. What scar tissue willdevelop in this case?

A. Healing without a scarB. Atrophic scarC. Hypertrophic scarD. Hypotrophic scarE. Keloid scar

66. A 22-year-old patient has suffered uni-lateral linear fracture in the area of the goni-al angle. Immobilization was provided with fulldental brace with loops and intermaxillary elasticexpansion. Recovery was uncomplicated. Thebrace should be removed after:

Page 18: Krok 2 - testcentr.org.ua · D. Monoblastic leukemia E. Acute herpetic stomatitis 18. A 57-year-old retired man complains of attacks of burning pain and rashes on the skin of his

Хiрургiчна стоматологiя 18

A. 3 weeksB. 2 weeksC. 1 weekD. 10 daysE. -

67. A 27-year-old patient was provisionally di-agnosed with acute suppurative odontogenicmaxillary sinusitis. What radiology methodwould be the most informative in this case?

A. Computed tomographyB. X-rayC. Panoramic radiographyD. Spot-film radiographyE. -

68. A 22-year-old man presents with swollen andhyperemic mucosa of the retromolar area; tooth38 is covered with hood-shaped gingival flap thatdischarges pus on palpation; body temperatureis 37.5oC. What urgent aid should be given to thepatient in this case?

A. Gingival flap incision and antibacterialtreatmentB. Gingival flap excisionC. Extraction of tooth 38D. Antibiotic treatmentE. Gingival flap incision

69. A 34-year-old man complains of pain inthe area of his right eye, headache, and bodytemperature rise up to 38,6oC. Two days ago thepatient developed an infiltration in the lowereyelid of the right eye. Objectively the eyeli-ds are markedly swollen, palpebral fissure isclosed, conjunctiva is swollen. Exophthalmosis observed. The eyeball is immobile, vision isimpaired. Make the diagnosis:

A. Orbital phlegmonB. Eyelid phlegmonC. Purulent maxillary sinusitisD. Angular vein trombophlebitisE. Lower eyelid abscess

70. A 43-year-old man came to the maxillofaci-al surgeon with complaints of aesthetic defect.Examination revealed excessive accumulation ofadipose tissue in the patient’s neck and uppertorso, which resembles collar with unclear magi-ns; neck mobility is reduced. Family historyshows the same symptoms to be present in thepatient’s father. Make the provisional diagnosis:

A. Madelung’s deformityB. LipomaC. NeurofibromatosisD. FibromaE. Lymphangioma

71. A 73-year-old man is registered for regularcheck-ups in an oncological clinic after completi-on of the combined treatment for oral mucosacancer stage II (radiation therapy and surgery).During one of the routine check-ups an areaof exposed mandibular bone is detected. Thereare no inflammatory changes of surroundingmucosa. A fistula tract with soft granulation isdetected. Mandibular X-ray shows a sequestrumwithout clear margin between healthy andnecrotic bone. What is the most likely provisi-onal diagnosis?

A. Mandibular osteoradionecrosisB. Acute purulent mandibular osteomyelitisC. Posttraumatic mandibular osteomyelitisD. Relapse of oral mucosa cancerE. Chronic mandibular periostitis

72. A 35-year-old patient complains of burns ofthe face and neck, swelling and burning painin the affected area. On examination: edema ofthe face and neck, palpebral fissure is narroweddue to swelling, affected skin is hyperemic andcovered with strained thin-walled blisters filledwith clear content. Where blisters are broken,there are pink wounds, sharply painful to touch.Determine the degree of the burns:

A. IIB. IC. III AD. III BE. IV

73. A 19-year-old patient came to a dentofacialclinic with complaints of pain in the gonial angleon the right, impaired mouth opening and pai-nful chewing. The signs had been persisting for5 days, emerged spontaneously and had beenaggravating gradually. Mandibular contractureis of the III degree. On examination of theoral cavity: hyperemia, edema of the retromolarspace on the right, hood-shaped mucosa fromunder which pus is being discharged and 2 toothtubercles can be detected. X-ray shows obliquemedial tooth position. Make the diagnosis:

A. Acute suppurative pericoronitis of the 48 toothB. Acute suppurative periostitis from the 48 toothC. Chronic local mandibular osteomyelitisD. Mandibular angle fractureE. Acute submandibular sialadenitis

Page 19: Krok 2 - testcentr.org.ua · D. Monoblastic leukemia E. Acute herpetic stomatitis 18. A 57-year-old retired man complains of attacks of burning pain and rashes on the skin of his

Ортопедична стоматологiя 19

1. A 27-year-old man complains of teeth mobi-lity in his upper and lower jaws. Objectively:dentition is intact. Central occlusion is determi-ned. What examination methods should be appli-ed?

A. Analysis of diagnostic models of the jawsB. GnathodynamometryC. MasticatiographyD. X-rayE. Electromyography

2. A 47-year-old man complains of mobility of theartificial crown on the 36 tooth, which was made 2years ago. Objectively: the 36 tooth is covered wi-th full metal swaged crown. Crown decementationand dentin demineralization are observed. Whatis the cause of such complication?

A. The crown is loose at the tooth cervixB. The crown edge is embedded into the gingivalpocketC. There are interdental contactsD. The crown contacts with antagonistic teethE. Useful life of the crown is exceeded

3. A removable full denture for the lower jaw isbeing made for a 75-year-old man. Objectively thealveolar process is slightly atrophied. Herbst testsare performed during fitting of an impression tray.When lips are stretched forwards the tray slips off.Where should the tray edge be shortened in thiscase?

A. From canine to canine on the vestibular sideB. From canine to canine on the lingual sideC. From behind the mandibular tuberosity to themylohyoid lineD. Along the mylohyoid lineE. In the premolar area on the lingual side

4. A 19-year-old woman, an actress, complai-ns of discoloration of her left maxillary centralincisor. One year ago the pulp of this tooth wasremoved and the tooth was filled. Gradually thetooth assumed grayish color. Objectively the 11is filled, discolored, firm, painless on percussion.Deep occlusion is observed. What part of the cli-nical presentation contraindicates installation ofan all-porcelain crown?

A. Deep occlusionB. Front teeth defects that cannot be correctedwith fillingsC. Enamel hypoplasia with tooth deformation anddiscolorationD. Tooth discolorationE. Devitalized teeth defects that cannot becorrected with dental inlays

5. A 32-year-old woman needs a denture. Onobjective examination the decision was made infavor of porcelain-fused-to-metal crown. Whatmaterial should be used in this case to obtain theimpression?

A. StomaflexB. RepinC. StomalginD. StensE. Orthocor

6. A 18-year-old woman needs a denture. Objecti-vely: the 21 tooth is dark gray in color, devitalized;

orthognathic occlusion is observed. The tooth isto be covered with plastic crown. What plasticshould be used to make the crown?

A. Sinma-MB. PhtoraxC. Protacryl-MD. BacrylE. Etacryl

7. A 58-year-old patient has made an appointmentto make a denture. Objectively: the 22 tooth is fi-rm and intact. The alveolar crest is atrophied; thepalate is flat. Removable denture is to be made.What approach regarding the 22 tooth should bechosen by a dentist?

A. To make a telescopic crownB. To retain the toothC. Removal of tooth pulpD. Tooth extractionE. To make a stump crown

8. A 24-year-old woman has Richmond crown bei-ng made to restore the crown of the central maxi-llar incisor. The cap is completed. What is the nextstep of prosthesis-making?

A. To fit the cap on the tooth stump and place thepost in the root canalB. To solder the post with the capC. To fit the cap and the post to the tooth rootD. Making of combination dental crownE. Tooth fixation with cement

9. A 53-year-old patient complains of pain andclicking in the left temporomandibular joint.Objectively: the face is symmetrical, palpationof the lateral pterygoid muscles is painful on theleft side. Mouth opening is reduced. Tomographyshows smooth bone outline of joint surfaces. Whi-ch disease of those listed below corresponds withthis clinical presentation?

A. Temporomandibular joint dysfunctionB. Rheumatic arthritisC. Deforming arthrosisD. Acute posttraumatic arthritisE. Joint ankylosis

10. A 47-year-old man complains of partial lossof his upper teeth. The patient’s medical hi-story states loss of teeth due to trauma sustai-ned 3 months ago. 11 and 12 are lost. 13, 21,and 22 are destroyed by 2/3 and restored withfillings. Occlusion is orthognathic. What dentureconstruction would be optimal for this patient,considering his occupation as a lecturer?

A. Porcelain-fused-to-metal dental bridgeB. Plastic dental bridgeC. Clasp-retained (bugel) removable partialdenture with attachmentsD. Removable partial laminar denture for theupper jawE. Swaged-soldered metal dental bridge withfaceted intermediate part

11. A 32-year-old man has metallic inlay made forhim. The denture is being made for tooth 36 withBlack’s class I carious cavity. What surfaces of theinlay should be filed down and polished beforefixing the denture?

Page 20: Krok 2 - testcentr.org.ua · D. Monoblastic leukemia E. Acute herpetic stomatitis 18. A 57-year-old retired man complains of attacks of burning pain and rashes on the skin of his

Ортопедична стоматологiя 20

A. Occlusal surfaceB. Lateral surfacesC. All surfacesD. Inlay bottomE. Lateral surfaces and inlay bottom

12. A 47-year-old patient presents with roundedbone protrusions 0,7-0,8 cm in size on theinner surface of the edentulous mandible in thepremolar area. The denture for this patient shouldhave:

A. Elastic linerB. Kemeny claspsC. Metal baseD. Orifices for the exostosesE. Dentogingival clasps

13. A 78-year-old patient is completelyedentulous. He has been wearing dentures for19 years. The patient complains of poor fixati-on of the upper denture. Objectively: the lowerthird of face is shortened, the alveolar processesof both jaws are markedly atrophied, the palate isflat. Mucous membrane in the denture-supportingarea is atrophied. How often should the denturesbe remodelled or restored?

A. Every 3-4 yearsB. Every 6 monthsC. Once a yearD. Every 7 yearsE. Every 10-12 years

14. A 43-year-old woman complains ofmobility and displacement of her upperfront teeth. Objectively: dental formula is

17 16 15 14 13 12 11 21 22 23 24 25 26 2747 46 45 44 43 42 41 31 32 33 34 35 36 37 .

Teeth 12 11 21 22 are slanted towardsthe vestibular side, diastema and tremata areobserved, I-II degree teeth mobility is detected.Select the orthodontic appliance for correction ofteeth misalignment as a part of complex treatmentof periodontal disease:

A. Palatal plate with vestibular archB. Bynin applianceC. Schwartz applianceD. Katz crownE. Palatal plate with inclined plane

15. A 45-year-old man complains of toothacheand mobility of his upper front teeth. Objectivelyhis dental formula is as follows:

17 16 15 14 13 12 11 21 22 23 24 25 26 2747 46 45 44 43 42 41 31 32 33 34 35 36 37 .

Dental cervices of 13 12 11 21 22 are exposedand demonstrate mobility of the III degree.Mobile teeth are to be extracted and immediatedenture is to be made for the patient. How soonafter the teeth extraction should such dentures beinserted?

A. On the day of teeth extractionB. In 1-2 daysC. In 3-4 daysD. In 5-6 daysE. In 6-7 days

16. When a prosthodontist was preparing the pati-ent’s tooth, the patient had epileptic seizure thatwas subsequently terminated. What mistake hadbeen made by the doctor?

A. No inquire into the patient anamnesisB. No inquire into the antecedent anamnesisC. No anaesthesiaD. Crude preparationE. Did not decline the appointment

17. A 35-year-old man came to the prosthodonticclinic with complaints of teeth mobility on hislower jaw. What type of occlusion stabilization isrecommended in this case?

A. ArchB. SagittalC. FrontalD. ParasagittalE. Frontosagittal

18. A 57-year-old patient complains of toothmobility and inability to eat. Objectively: thelower 35, 36, 37, 38, 44, 45, 46 and 48 teeth aremissing; the 31, 32, 33, 34, 41, 42, 43, 47 teethexhibit grade II mobility, their clinical crowns arelow, tooth equator is not pronounced. What is theoptimal denture construction in this case?

A. Removable cast splintB. Removable partial dentureC. Kurlyandsky splint barD. Bynin removable splintE. Removable splint with vestibulo-oral clasp

19. A 38-year-old patient with chronic generalizedperiodontitis has been referred for orthopedictreatment. Objectively: dentitions are withoutgaps, the 12, 11, 21, and 22 teeth are pulplessand exhibit I grade mobility. The other teeth arefirm. What is the most aesthetic dental splint forthe anterior teeth?

A. Mamlok’s splintB. Ring splintC. Soldered combined crownsD. Cap splintE. Mouthguard

20. The 40-year-old woman complains of inabili-ty to properly masticate due to the loss of thefollowing lateral teeth: 18, 16, 15, 25, 26, 28, 38,35, 36, 44-46, and 48. The rest of her teeth presentwith the I-II degree of mobility. Generalized peri-odontitis is observed. What denture constructionwould be optimal in this case?

A. Removable dental splintB. Clasp-retained (bugel) removable partialdentureC. Removable laminar dentureD. Fixed dental bridgeE. Metal-based denture

21. A 55-year-old patient requires a denture.Objectively: Kennedy’s I class dentition defect;the 16, 17, 18, 26, 27, and 28 teeth are missi-ng. The patient presents with fixed occlusion.The 15 and 25 teeth have low crowns with pooranatomic contours, intact. Clasp-retained (bugel)removable partial denture is being made for thepatient. What fixation system would be optimal inthis case?

Page 21: Krok 2 - testcentr.org.ua · D. Monoblastic leukemia E. Acute herpetic stomatitis 18. A 57-year-old retired man complains of attacks of burning pain and rashes on the skin of his

Ортопедична стоматологiя 21

A. Telescopic fixationB. AttachmentsC. Roach clasp (clammer)D. Aker-Roach combined clasp (clammer)E. Continuous clasp (clammer)

22. A patient needs a clasp-retained (bugel)removable partial denture. It is planned to studythe jaw model by means of a parallelometerin order to determine the required depth ofthe undercuts on the abutment teeth. Specifythe length of the measuring rods used for thispurpose:

A. 0,25 0,50 0,75B. 0,15 0,40 0,65C. 0,20 0,45 0,70D. 0,30 0,55 0,80E. 0,35 0,60 0,85

23. A 30-year-old man presents with fresh medianmandibular fracture without visible displacementof the fragments. What will be the function of thedental apparatus to be prescribed in this case?

A. FixationB. SettingC. DirectingD. ReplacementE. Formation

24. A 43-year-old woman complains of her lowerteeth mobility. Objectively the teeth mobility is ofthe I-II degree. It is planned to make a full-castremovable occlusal splint for her. What materialwould be optimal for this splint?

A. Cobalt nickel chromium alloyB. ”EI-95” alloyC. Stainless steelD. ”PD-250” alloy (silver palladium alloy)E. Gold alloy of 900 millesimal fineness

25. A 40-year-old man presents with pathologicteeth grinding caused by their functional overloaddue to the loss of many antagonist teeth. With di-rect occlusion, vertical grinding of the front teethresulted in protrusion of the patient’s lower jawforwards. Interalveolar space is diminished, thelower third of the face is shortened. What wouldbe the most advisable treatment in this case?

A. Prosthetics that increase interalveolar heightB. Prothetic treatmentC. Teeth shorteningD. Prosthetic treatmentE. Instrumental surgical treatment

26. In a prostodontic clinic a partial laminardenture for the upper jaw is being made for a 53-year-old patient. Objectively: dental formula is14, 13, 12, 11, 21, 22, 23, 24, 27. The teeth are firm,clinical crowns are tall with pronounced equator.X-ray shows no periapical changes in the peri-odontium of the abutment teeth. What clasp fi-xation is optimal for this patient?

A. PlanarB. SagittalC. DiagonalD. TransversalE. Point

27. A 20-year-old man complains of missing tooth

on the upper right jaw, aesthetic defect. Objecti-vely: the 12 tooth is absent, adjacent teeth areintact, firm, with distinct anatomical shape andtall crowns. Direct occlusion is observed. Duri-ng the interview the patient was found out tohave congenital heart disease. What dentureconstruction would be optimal in this case?

A. Adgesive dental bridgeB. Plastic dental bridgeC. Porcelain-fused-to-metal dental bridge with 14and 13 abutment teethD. Plastic-fused-to-metal dental bridgeE. Swaged-soldered dental bridge

28. A 48-year-old patient complains of the lowerjaw teeth mobility. Van Thiel dental splint isto be made for prosthodontic treatment. Whatconstruction elements are supposed to fix it inplace?

A. Whole piece proximal grip claspsB. Full metal crownsC. Wire claspsD. Parapulpar postsE. Equator crowns

29. A 55-year-old man suffered a blow to thefrontal mandibular area. He is diagnosed wi-th mandibular fracture. Prior to trauma he waswearing removable dentures (partial laminardenture for the lower law and full denture forthe upper jaw). What can be used for transportimmobilization of the fracture?

A. Patient’s denturesB. Weber splintC. Vankevych splintD. Zbarzh apparatusE. Vasiliev splint

30. A cast clasp-retained (bugel) removable parti-al denture is being made for a 58-year-old pati-ent. Impressions are made, centric jaw relation isdetermined, plaster casts are obtained. What isthe next stage?

A. Examination of the working model with aparallelometerB. Transfer of denture frame pattern to the worki-ng modelC. Wax modelling of the denture frameD. Duplication of the working modelE. Marking the border seal

31. On objective examination a 59-year-old manwith the edentulous mandible presents with boneprotrusions and mobile areas of the alveolar crest.To ensure proper fixation of the denture andeven load distribution the following functionalimpression should be made:

A. DifferentiatedB. Complete anatomicalC. CompressionD. DecompressionE. Combined

32. A patient with post-resection upper jawdefect that invades the nasal cavity has cometo a prosthodontic clinic. What denture isrecommended in the given case?

Page 22: Krok 2 - testcentr.org.ua · D. Monoblastic leukemia E. Acute herpetic stomatitis 18. A 57-year-old retired man complains of attacks of burning pain and rashes on the skin of his

Ортопедична стоматологiя 22

A. Replacement denture with obturating elementB. Floating obturatorC. MouthguardD. Forming dentureE. Replacement denture

33. A 25-year-old man complains of incorrectlypositioned maxillary left central incisor due totrauma sustained 2 months ago. Objectively tooth21 is rotated around its axis into palatal positi-on. What would be the most advisable treatmentmethod for correction of this defect?

A. Orthodontic treatmentB. Surgical treatmentC. Instrumental surgical treatmentD. -E. Splinting followed by prosthetic treatment

34. A 62-year-old patient came to a dental cli-nic with complaints of facial swelling, pain in thelower left jaw, and numb lower lip. On clinicalexamination he was diagnosed with fracture ofthe body of mandible on the left, edentulous jaws,microstomia. Choose the optimal construction:

A. Limberg’s dental splintB. Weber’s dental splintC. Guning-Port’s dental splintD. Elbrecht’s dental splintE. Vankevych dental splint

35. A 48-year-old patient came to a dentist afterthe maxillectomy on one side conducted 3 daysago. Remaining teeth are firm. Treatment planforesees making an Oxman’s denture for the pati-ent. What part of the denture should be producedfirst?

A. FixatingB. ObturatingC. ResectionD. FormingE. Substituting

36. A 46-year-old patient complains of masticati-on disorder caused by the lack of the 34, 35, and36 teeth. The antecedent anamnesis is as follows:the teeth were extracted 3 months ago due tocomplication of cariosity. The patient anamnesis:the history of tonsillitis, rheumatoid arthritis andBotkin’s disease. After the appointment with thispatient the instruments should be sterilized in thefollowing way:

A. Specialized procedureB. Dry-heat sterilizerC. Processing with lysolD. Processing with 0,1% chloramine solutionE. General procedure

37. A man complains of gingival pain in his upperleft jaw and bleeding that occurs when he brusheshis teeth or eats solid food. Objectively on theupper jaw he wears a swaged-soldered metalbridge with 14 and 16 as abutment teeth. Thecrown edge is pushed under the gingival marginby 0.3 mm. Intermediate part is closely fittedto the gums. The mucosa is hyperemic, swollen,interdental papillae are smoothed out; touchingmucosa with a dental instrument provokes bleedi-ng. What medical tactics should the dentist choosein the first place?

A. Remove the dental bridgeB. Refer the patient for cosultation with the dentaltherapistC. Refer the patient for X-rayD. Refer the patient for clinical blood testE. Make metal-fused-to-porcelain dental bridge

38. When checking construction of the soldereddental bridge with the 35 and 38 abutment teeththe following was detected: pores in the placewhere abutment crowns and intermediate part aresoldered together; masticatory cusps are sharplydefined; there is early contact with antagonistteeth; the intermediate part makes tight contactwith the alveolar process mucosa. How can thoseflaws be corrected?

A. Dental bridge should be remadeB. Intermediate part should be correctedC. Masticatory surface should be corrected, andsoldered places - polishedD. Masticatory surface should be corrected, andthe height of the flushing part is to be increased upto 2 mmE. Tooth-antagonists should be shaved off

39. Due to lack of timely specialized treatment a44-year-old man presents with incorrectly healeddisplaced mandibular fracture. Objectively thelower jaw narrows sharply, vestibular cusps of thelower teeth contact with oral cusps of the upperteeth. The patient declined surgical treatment.What treatment tactics should be chosen by thedentist in this case?

A. Make a prosthesis with double dentitionB. Make a non-removable dental bridge withmovable jointC. Correct the malocclusion by filing down thepatient’s teethD. Correct the malocclusion via instrumentalsurgical methodE. Make a dentogingival laminar denture

40. A 57-year-old man presents with habitualmandibular dislocation. To reduce mouth openi-ng, Yadrova apparatus was made. How longshould the treatment last in this case?

A. 3 monthsB. 6 monthsC. 9 monthsD. 12 monthsE. 18 months

41. An HIV-infected patients needs a dentalprosthesis. The dentist plans to make dental bri-dges for this patient. How should the instrumentsbe processed after the appointment?

A. According to the special schemeB. According to the usual schemeC. In a hot air sterilizerD. With lysoforminE. With 3% chloramine solution

42. A 30-year-old man complains of pain in hisfront lower teeth, which he attributes to a traumato the mental region. Objectively: continuousdentition, orthognathic occlusion. X-ray showsa median mandibular fracture. What dental splintwould be optimal?

Page 23: Krok 2 - testcentr.org.ua · D. Monoblastic leukemia E. Acute herpetic stomatitis 18. A 57-year-old retired man complains of attacks of burning pain and rashes on the skin of his

Ортопедична стоматологiя 23

A. Flat occlusal splintB. Soldered splint on ringsC. Cap splintD. Weber’s splintE. Plastic mouthguard

43. A 27-year-old woman complains of recurrentloss of a tooth filling in the lower right jaw.Objectively: in the 46 tooth on the masticatoryapproximal surface there is a defect of hard toothtissues affecting 1/3 of the tooth crown, no toothdiscoloration; positive, quickly abating reacti-on to cold stimulus is observed. What dentureconstruction would be optimal in this case?

A. Dental inlayB. Combined crownC. Porcelain-fused-to-metal crownD. Plastic crownE. Partial crown

44. To make the external prosthesis for a 62-year-old man it is necessary to obtain a Hippocratesfacial moulage of this patient. What impressionmaterial should be used?

A. PlasterB. DentafolC. StensD. StomaflexE. Repin

45. A 60-year-old patient has been undergoing theprocedure of checking the complete removabledentures construction and fixing teeth on waxbases. The following flaws have been detected:fissure between the teeth on the frontal area andcusp-to-cusp contact in the lateral area. What mi-stake had been made?

A. Anterior occlusion had been determinedinstead of central oneB. Posterior occlusion had been determinedinstead of central oneC. Lateral occlusion had been determined insteadof central oneD. Models had been plastered in a wrong way inan occluderE. Swabs had been crushed during determinationof central occlusion

46. Dental splint is being designed in aprosthodontics clinic for a 39-year-old pati-ent with generalized periodontitis, II degree.Gnathodynamometer is used to measure theperiodontium load resistance. What anatomico-functional data are obtained with this method?

A. Masticatory pressureB. Masticatory forceC. Periodontium pliancyD. Masticatory muscles toneE. Masticatory efficiency

47. A 45-year-old man complains of impairedchewing due to pathologic lower jaw mobility.The patient was diagnosed with false joint in thearea of absent 33 and 34 teeth. On X-ray: defectof the mandibular body is 0,8 cm in size. The teethon the fragments on both sides of the defect areintact. What denture would be recommended inthis case?

A. Oxman’s fixed dental bridgeB. Tigerstedt’s flat occlusal splintC. Weber’s dental splintD. Entin’s stiff head-chin strapE. Tigerstedt’s wire anchor splint

48. A 55-year-old man was delivered into thehospital with bilateral mandibular fracture withinthe dentition. Objectively teeth 34, 35, 36, 45, and46 are missing. Lower incisors are mobile (I-IIdegree). Fragment displacement is insignificant.What splint should be made for this patient?

A. WeberB. VankevychC. TigerstedtD. Flat occlusal splintE. Gunning-Port

49. A 45-year-old patient is prescribed a dentalbridge supported with implants. In the processof preparation to the prosthodontic treatmentthere were intraosseous screw two-stage implantsplaced in the area of the 34 and 36 teeth. Howlong is the period necessary for implant integrati-on in this case?

A. 3 monthsB. 2 weeksC. 6 monthsD. 10 monthsE. 1 year

50. A 70-year-old man has edentulous maxilla.Objectively maxillary tuberosity and alveolarprocesses are completely atrophied; palatine vaultis flat, its mucosal layer is moderately pliant. Inthis case the patient’s atrophic edentulous maxillacan be classified as:

A. Schroeder class IIIB. Keller class IIIC. Schroeder class IID. Keller class IIE. -

51. A laminar denture for the lower jaw is bei-ng made for a 54-year-old patient. Base platewax is used during the laboratory stage for waxtemplates. What group of accessory materialsdoes such wax belong to?

A. ModelingB. AbrasiveC. FixingD. ImpressionE. Forming

52. The medical station of a regiment received apatient with signs of bilateral mandibular fracture.What is the main task of first aid in this case?

A. To control shock, bleeding, and asphyxia and toprovide transport immobilizationB. To check and correct previously appliedbandagesC. To administer analgesics and cardiac medicati-onsD. To clean oral cavity from blood clots, toothshards, and bone fragmentsE. To provide symptomatic therapy and care

53. A 55-year-old man came to the prosthodonticclinic to have a denture made for him. Tooth 11is missing in the patient. Two days ago he was

Page 24: Krok 2 - testcentr.org.ua · D. Monoblastic leukemia E. Acute herpetic stomatitis 18. A 57-year-old retired man complains of attacks of burning pain and rashes on the skin of his

Ортопедична стоматологiя 24

released from the in-patient unit after a case ofmyocardial infarction. What tactics should thedentist choose?

A. Make a temporary removable dentureB. Make a clasp-retained (bugel) removable partialdentureC. Make a dental bridge with 12 and 21 asabutment teethD. Temporarily refrain from making a dentureE. Perform implantation

54. A 45-year-old patient came to a prosthodonti-cs clinic. During the objective examination thedoctor checked the sagittal movements of thelower jaw. What muscles are responsible for sagi-ttal movements of the lower jaw?

A. Lateral pterygoid musclesB. Medial pterygoid musclesC. Mandibulohyoid muscleD. Digastric muscleE. Mentohyoid muscle

55. A 43-year-old patient complains of mobi-lity and significant neck exposure of the lowerfront teeth. Objectively: the gums in the area ofthe 44, 43, 42, 41, 31, 32, 33, and 34 teeth arepale and cyanotic, non-bleeding. The 42, 41, 31,and 32 teeth exhibit the I-II grade mobility. Theovercrowding of the 42, 41, 31, and 32 teeth ispresent. The necks of the 42, 41, 31, and 32 teethare exposed by 1/2 of the root length, the necks ofthe 43 and 33 teeth are exposed by 1/4. What kindof denture should be applied in this case?

A. Cast removable splintB. Kurlyandsky’s bar splintC. Cap splintD. Partial crownE. Half-ring splint

56. Removable partial dentures for upper andlower teeth are being made for a 45-year-old man.Complete anatomical impressions were made usi-ng ”Ypeen” alginate material. What should beused for disinfection of obtained impressions?

A. 2.5% glutaraldehyde with рH- 7.0 - 8.7B. 0.1% desoxone solutionC. -D. Phenol solution in proportion 1:20E. 6% hydrogen peroxide solution

57. A 27-year-old man presents with missing crownof 11. Objectively teeth 21 and 12 are intact;intraoral spot film X-ray shows the root of 11 tobe filled to the apex, no changes in the periapicaltissues, no pathologic mobility. What constructionof the denture should be recommended for thispatient?

A. Metal stump inlay with overlaying porcelain-fused-to-metal crownB. Stump inlay with overlaying swaged crownC. Stump inlay with overlaying plastic crownD. Stump inlay with overlaying full cast metalcrownE. Remove the root of 11 and perform implantati-on

58. A 54-year-old patient complains of frequentcrunching sound in the right temporomandibularjoint, which developed one month ago. In the

morning the crunching is more frequent anddecreases towards the evening. Objectively: theface is symmetrical, the skin over the joint is wi-thout changes, the mouth opens by 2.9 mm. Whatis the most likely diagnosis in this case?

A. ArthrosisB. Acute arthritisC. Temporomandibular joint dislocationD. Chronic arthritisE. Temporomandibular joint pain dysfunctionsyndrome

59. A 63-year-old man complains of pain in thearea of maxillary mucogingival fold caused by usi-ng a removable laminar denture. Objectively: inthe area of the mucogingival fold there is a trophiculcer with swollen margins and hemorrhagingfloor. Make the diagnosis:

A. Denture-related stomatitisB. Toxic chemical stomatitisC. Toxic infectious stomatitisD. Allergic contact stomatitisE. Greenhouse effect

60. A 64-year-old man complains of pain andmobility of his front lower teeth. Objectively: the43, 42, 41, and 31 teeth demonstrate the III degreemobility. These teeth are planned to be extractedand immediate-insertion denture is to be made.When should this denture be put in place?

A. On the day of the teeth extractionB. In 2-4 days after the teeth extractionC. In 6-8 days after the teeth extractionD. In 3-6 days after the teeth extractionE. In 1-2 days after the teeth extraction

61. A 36-year-old woman needs a dentalprosthesis. Objectively there is a carious cavityon the mesial masticatory surface of tooth 46,interdental contact is disturbed. Dental inlay isto be made for this patient. According to Black’sclassification of dental caries this cavity is class:

A. 2B. 3C. 4D. 5E. 1

62. A 45-year-old man complains of pain andcrepitation in the temporomandibular joint duri-ng the movements of the lower jaw. Objectively:the face is symmetrical, the mouth opens with sli-ght displacement to the left. Dentition is intact. Toclarify the diagnosis X-ray of the temporomandi-bular joint was performed. Where should theheads of the mandible be located normally duringmaximum mouth opening?

A. At the top of the articular tubercleB. In the center of the glenoid fossaC. In front of the articular tubercleD. In the center of the articular tubercleE. Closer to the distal part of the glenoid fossa

63. A 45-year-old man complains of pain andcrepitation in the temporomandibular joint duri-ng the movements of the lower jaw. Objectively:the face is symmetrical, the mouth opens with sli-ght displacement to the left. Dentition is intact.On occlusiography there were detected centric

Page 25: Krok 2 - testcentr.org.ua · D. Monoblastic leukemia E. Acute herpetic stomatitis 18. A 57-year-old retired man complains of attacks of burning pain and rashes on the skin of his

Ортопедична стоматологiя 25

and eccentric supracontacts. What treatmentmethods should be applied in the first place?

A. Selective teeth shavingB. Mouthguard for muscle relaxationC. Appliances that limit mouth openingD. Mouthguards that increase the height of centralocclusionE. Lower jaw immobilization

64. A 45-year-old patient complains of inability toproperly masticate due to the loss of lateral teeth.The 17, 16, 15, 25, 26, 27, 37, 36, 35, 44, 45, and 46teeth are missing. The retained teeth exhibit theI-II degree of mobility. The patient is diagnosedwith generalized periodontitis. Kennedy class Identition defects are observed. What constructi-on would be optimal in the given case?

A. Clasp-retained (bugel) removable partialdenture with splinting elementsB. Partial laminar dentureC. Elbrecht’s dental splintD. Mamlok’s dental splintE. Cantilever dental bridges

65. A 70-year-old patient addressed a hospitalwith complaints of poorly stabilized completeremovable dentures of the upper and lower jaws.What method of artificial teeth arrangementis preferable in making of a new completeremovable denture?

A. According to individual occlusal curvesB. According to disocclusal planesC. According to standard occlusal curvesD. According to spherical occlusal curvesE. According to prothetic occlusal planes

66. A 45-year-old woman needs a denture.Objectively: the 17, 16, 15, 14, 12, 25, and 26 teethare missing. Specify the Kennedy’s class of denti-tion defects in the given case:

A. II class, 2 subclassB. II class, 4 subclassC. III class, 1 subclassD. III class, 3 subclassE. II class, 3 subclass

67. A 46-year-old man complains of constant losi-ng of a filling in his lower right tooth. Objectively:in the 16 tooth on the approximal masticatorysurface there is a defect of crown hard tissues at1/3. The tooth has no discoloration, percussionis painless. What construction should be prescri-bed?

A. InlayB. Porcelain-fused-to-metal crownC. Semi-crownD. 3/4 crownE. Plastic crown

68. A 37-year-old patient complains of anaesthetic defect. Objectively: the 13 tooth isdestroyed by 2/3. The tooth is pulpless, the rootcanal is filled. How deep should the root canal beopened for pivot crown installation in this pati-ent?

A. 2/3 of the root canalB. 1/3 of the root canalC. 3/4 of the root canalD. 1/2 of the root canalE. Full length of the root canal

69. A 68-year-old man has removable partialdentures made for him. At the stage of checki-ng the denture construction, occlusal contact inthe lateral areas is cusp-to-cusp and frontal areahas sagittal fissure. What mistake was made by thedoctor?

A. Anterior occlusion is fixedB. Lateral occlusion is fixedC. Prothetic plane is designed incorrectlyD. Height of centric occlusion is understatedE. Height of centric occlusion is overstated

70. A 46-year-old man, a teacher, complains oflower teeth mobility that impedes the process ofbiting. Objectively the dentition is uninterrupted,front teeth demonstrate the II degree of mobility.X-ray shows straight and filled root canals of 32,31, 41, and 42. What appliance will stabilize frontteeth while retaining their aesthetic appearance?

A. Mamlok’s dental splintB. Removable segmented splint for the front teethC. Cap splintD. Splint with embrasure claspsE. Semicrown splint

71. A clasp-retained (bugel) removable partialdenture for the lower jaw is to be made for a 53-year-old patient. Objectively: the 38, 37, 35, 34, 45,46, and 47 teeth are missing. The retained teethare firm, with low clinical crowns. What fixationmethod of the denture would be optimal in thiscase?

A. Telescopic systemB. Supporting-retaining clasps (clammers)C. Bar systemD. AttachmentsE. Ball joint attachments

72. A 40-year-old patient complains of pain inthe tragus area, clicking during mouth opening,stuffed ears. Objectively the face is symmetrical,mouth opening path is straight. Dentition defectcan be estimated as the I class by Kennedy; the18, 17, 16, 26, 27, 28 teeth are missing. In this casethe load would be the most traumatizing for thefollowing anatomical structure:

A. Interarticular diskB. Articular capsuleC. Articular headD. Distal slope of the articular tubercleE. Floor of the temporal bone socket

73. A 50-year-old man complains of bared dentalcervices on his upper and lower jaws. Objectively:the teeth and dentition are intact, clinical crownsare elongated, the teeth have no pathologic mobi-lity, are worn off within the physiological norm.To remove supracontacts it is planned to performselective teeth shaving. What additional investi-gation is necessary in the given case?

A. OcclusiographyB. X-rayC. MasticatiographyD. GnathodynamometryE. Mastication tests

Page 26: Krok 2 - testcentr.org.ua · D. Monoblastic leukemia E. Acute herpetic stomatitis 18. A 57-year-old retired man complains of attacks of burning pain and rashes on the skin of his

Дитяча терапевтична стоматологiя 26

1. A 18-year-old boy complains of bleedi-ng and pain in her gums. The disease onsetwas 4 days ago. Objectively the skin is pale,body temperature is 38.5oC . Her submandibularlymph nodes on the left are enlarged, painful,non-fused with the surrounding tissues. The gi-ngival papillae and gingival margin in the area of33, 34, 35, 36, and 37 are hyperemic, ulcerated,and covered in necrotic deposit. Teeth presentwith soft dental plaque. Make the diagnosis:

A. Necrotizing ulcerative gingivitisB. Acute catarrhal gingivitisC. Chronic hypertrophic gingivitisD. Chronic catarrhal gingivitisE. -

2. Parents of a 6-month-old child complain oftheir child having a large amount of dentaldeposit in the oral cavity. Objectively the oralmucosa is hyperemic, covered in white easilyremoved deposit that resembles curdled milk.What is the causative agent of this disease?

A. Candida fungiB. Herpes simplex virusC. Klebs-Loeffler bacillusD. Coxsackie virusE. Epstein-Barr virus

3. A 12-year-old boy complains of painful andbleeding gums on his upper jaw. Objectively thegingival margin in the area of the 13, 12, 11, 21,22, and 23 teeth is swollen, hyperemic, deformeddue to gingival overgrowths. Gingival papillaecover the crowns by 1/3 of their height, bleed ontouch. Upper front teeth are overcrowded. X-ray shows no pathological changes of the peri-odontium. What drugs should be administeredfor topical treatment in the first place?

A. Nonsteroidal anti-inflammatory drugsB. SclerosantsC. Steroidal anti-inflammatory drugsD. Keratoplastic agentsE. Cytostatic agents

4. Parents of a 3-year-old child report that thechild suffers from constant pain in the upperfront teeth. Objectively: the coronal part of the61 tooth is gray and decayed. Probing of theroot canal orifice is painful and accompanied bybleeding. The tooth percussion provokes acutepain. Mucosa is hyperemic, edematic and pai-nful. Palpation in the region of the 61 and 62teeth reveals a fistula. What is the provisionaldiagnosis?

A. Exacerbation of chronic periodontitisB. Acute suppurative periodontitisC. Acute diffuse pulpitisD. Chronic granulating periodontitisE. Exacerbation of chronic pulpitis

5. A 7-year-old boy complains of increased bodytemperature, up to 38oC , headache, sore throat.Objectively: there are erosions on the slightlyhyperemic mucosa of the soft palate, anteriorpalatal bars, and tonsils. The submandibularlymph nodes are slightly enlarged, painless.Name the causative agent of this disease:

A. Coxsackie virusB. Herpes simplex virusC. Epstein-Barr virusD. Klebs-Loeffler bacillusE. Bordet-Gengou bacillus

6. A 6-year-old boy complains of a cavity in theprevoiusly treated tooth. Objectively: in the 85tooth there is a carious cavity within mantledentin; the dentin is dense and pigmented;probing of the cavilty floor and walls, thermalstimuli, and percussion are painless. Sensitivityof the dentin-enamel junction is observed duri-ng the cavity preparation. Make the diagnosis:

A. Chronic median cariesB. Chronic deep cariesC. Acute median cariesD. Acute deep cariesE. Chronic superficial caries

7. A 12-year-old girl has complaint of a cari-ous cavity in her tooth. Objectively: there isBlack’s class 1 carious cavity in the 36 tooth;it is localized in the parapulpar dentin; themouth of the cavity is wide. The dentin is denseand pigmented. It is sensitive to cold stimulus,percussion is painless. What is the most likelydiagnosis?

A. Chronic deep cariesB. Chronic median cariesC. Acute deep cariesD. Acute median cariesE. -

8. A 10-month-old child is fussy, refuses to eat.Disease onset was 2 days ago. The child is beentreated by a pediatrician for pneumonia, recei-ves antibiotics and sulfanilamides. Objectively:the oral mucosa is hyperemic, swollen; there iswhitish coating on the mucosa of the cheeks,lips, soft and hard palate; coating removal cancause erosions. Submandibular lymph nodes areenlarged. What is the most likely diagnosis?

A. Acute candidal stomatitisB. Acute herpetic stomatitisC. Geographic tongueD. Allergic contact stomatitisE. Chronic candidal stomatitis

9. Parents of a 12-year-old child are concernedabout the child having white spots on the frontalteeth of the upper jaw; the spots appeared halfa year ago. Objectively: there are chalky spotsdetected in the cervical zone vestibular surfacesof the 13, 12, 11, 21, 22, and 23 teeth. The enamelin those spots is dull; probing revealed it to bepliant and coarse. The anamnesis states short-time pain caused by chemical stimuli. What isthe provisional diagnosis?

A. Acute initial cariesB. Chronic initial cariesC. Acute superficial cariesD. Systemic hypoplasia of enamelE. Dental fluorosis

10. A 14-year-old girl complains of bleedinggums and foul smell from her mouth. Objecti-vely: gingival mucosa is hyperemic, pastose,

Page 27: Krok 2 - testcentr.org.ua · D. Monoblastic leukemia E. Acute herpetic stomatitis 18. A 57-year-old retired man complains of attacks of burning pain and rashes on the skin of his

Дитяча терапевтична стоматологiя 27

hemorrhaging. Schiller-Pisarev test is positi-ve. Papillary marginal alveolar index is 70%.Fedorov-Volodkina Hygiene Index equals 3.X-ray of the frontal area of jaws demonstratesretained cortical plate. Make the diagnosis:

A. Chronic generalized catarrhal gingivitisB. Chronic generalized periodontitisC. Acute generalized catarrhal gingivitisD. Chronic generalized hypertrophic gingivitisE. Exacerbation of chronic generalized peri-odontitis

11. During regular check-up a 6.5-year-old childpresents with carious cavity on the distal proxi-mal surface of 65 within mantle dentin. Cavitywalls and bottom are pigmented, dense, pai-nless on probing; there is no response to coldstimulus; percussion is painless. During toothpreparation there is tenderness at the level ofdentinoenamel junction. What is the most likelydiagnosis?

A. Chronic median cariesB. Acute median cariesC. Chronic deep cariesD. Chronic fibrous pulpitisE. Chronic granulating periodontitis

12. A 4-year-old practically healthy child camefor oral cavity sanation. Objectively on themasticatory surface of 75 there is a carious cavi-ty within mantle dentin. The cavity is filled withsoftened dentin. Dentinoenamel junction is pai-nful on probing. What material would be opti-mal for permanent filling?

A. Glass ionomer cementB. Phosphate cementC. Silicophosphate cementD. Composite materialE. Silicate cement

13. A 7.5-year-old practically healthy childcomplains of crown fracture and pain in theupper right incisor. Objectively 2/3 of crown of11 is absent, the pulp is exposed and red; onprobing it is acutely painful and bleeding; toothpercussion is painful. The trauma occurred 2hours ago. What would be the optimal treatmentmethod in this case?

A. Vital amputationB. Devital amputationC. Vital extirpationD. Devital extirpationE. Biological approach

14. Parents of a 2.5-year-old child complain ofgradual destruction of the upper front teeth oftheir child for the last several months. Objecti-vely there are carious cavities within mantledentin on the contact and vestibular surfacesof 52, 51, 61, and 62. The cavities are filled wi-th softened pigmented dentin that can be easilyremoved with dental excavator. Make the provi-sional diagnosis:

A. Acute median cariesB. Chronic deep cariesC. Acute deep cariesD. Chronic median cariesE. Chronic superficial caries

15. A 7-year-old boy underwent fissure sealingin teeth 36 and 46. Fissure sealing would be mosteffective:

A. Immediately after eruption of the permanenttoothB. After permanent occlusion is formedC. If permanent teeth are affected with cariesD. In 1-2 years after tooth eruptionE. In 3-4 years after tooth eruption

16. A 6-year-old girl took paracetamol to treat acase of URTI two days ago, which resulted in thedevelopment of her present condition. The di-sease onset was acute with temperature increaseup to 39.8oC . Objectively there are cockade-shaped maculopapular rashes on her face.The vermilion border is swollen, hyperemic,covered in massive brown crusts, and presentswith bleeding cracks. Conjunctivitis is detected.Swollen and hyperemic oral mucosa presentswith numerous erosions covered with fibrinousincrustations; the erosions are sharply painfulon palpation. What is the most likely diagnosis?

A. Stevens-Johnson syndromeB. Erythema multiforme exudativumC. Acute herpetic stomatitisD. Chronic recurrent aphthous stomatitisE. Pemphigus

17. A 6.5-year-old child has closed non-pigmented fissures in the first permanent molar,which have been revealed during preventiveexamination. Enamel transparency is retained,its probing reveals no coarseness. Choose theoptimal method of treatment in this case:

A. Non-invasive sealingB. Invasive sealingC. Preventive fillingD. ART techniqueE. Regular medical check-ups

18. A 10,5-year-old child complains of painfulrash on his lips. Objectively: the red border ofthe lips is swollen, hyperemic, covered in fissuresand numerous scabs of dried blood. The skin ofthe upper lip has small blisters containing seroussubstance, which merge with each other in someplaces. Maceration and weeping skin also can beobserved, especially in the corners of the mouth.What is the most likely diagnosis?

A. Exematous cheilitisB. Meteorological cheilitisC. Atopic cheilitisD. Actinic cheilitisE. Exfoliative cheilitis

19. A 14-year-old teenager complains of dry andchapped lips especially in autumn and winter.Objetively the vermillion border is dry, infi-ltrated, and covered in numerous scales. Skin ofthe lips is dense, pigmented, with pronouncedpattern, peeling, and radial cracks. At the same

Page 28: Krok 2 - testcentr.org.ua · D. Monoblastic leukemia E. Acute herpetic stomatitis 18. A 57-year-old retired man complains of attacks of burning pain and rashes on the skin of his

Дитяча терапевтична стоматологiя 28

time the skin of the face is dry, lichenified, andexcoriated. What is the provisional diagnosis?

A. Atopic cheilitisB. Exfoliative cheilitisC. Actinic cheilitisD. Meteorological cheilitisE. Allergic contact cheilitis

20. A 13-year-old child complains of periodicalgingival bleeding during teeth brushing, whi-ch has been observed for half a year. Objecti-vely the gingival mucosa in the frontal mandi-bular area presents with congestive hyperemiaand edema. Decay-missing-filled (DMF) indexequals 4. Oral cavity hygiene is unsatisfactory.In this case it is necessary to recommend thepatient the toothpastes with:

A. Herbal extractsB. Zinc citrateC. Calcium glycerophosphateD. Amine fluoridesE. Salt additives

21. A 15-year-old girl complains of toothachethat persists for a day and increases on biting.Objectively: in the 36 tooth there is a deep cari-ous cavity non-communicating with the dentalcavity. No reaction to the thermal stimuli isobserved, probing of the carious cavity flooris painless. Vertical percussion is markedly pai-nful. Gingival mucosa in the area of the 36 toothis unaltered. X-ray presents with no alterations.Make the diagnosis:

A. Acute serous periodontitisB. Acute suppurative pulpitisC. Acute suppurative periodontitisD. Acute serous pulpitisE. Exacerbation of chronic periodontitis

22. Parents of an 8-year-old child complain ofrashes in the child’s oral cavity. Lately the chi-ld has been inert, refused to eat. On the oralmucosa there are small round erosions withclear margins. There are vesicles with turbidcontent on the child’s face and scalp. Make theprovisional diagnosis:

A. ChickenpoxB. MeaslesC. Hypertensive-hydrocephalic syndromeD. Infectious mononucleosisE. Scarlet fever

23. A 12-year-old girl complains of pain in hermouth that occurs during eating. According toher medical history these symptoms reemergeonce or twice per year. Objectively on themucogingival fold there are 3 aphthae 5-7 mm insize, they have yellowish coating and inflamedred border and are acutely painful on touch.Name the most likely diagnosis:

A. Chronic recurrent aphthous stomatitisB. Acute herpetic stomatitisC. Chronic recurrent herpetic stomatitisD. Erythema multiforme exudativumE. Toxic-allergic drug-induced stomatitis

24. A 5-year-old child complains of spontaneous

pain in an upper jaw tooth on the right thataggravates at night and during eating cold food.Objectively: the 65 tooth has a deep cavitycommunicating with the tooth cavity. Probingis painful, percussion is painless. Cold watercauses long-term pain. What is your provisionaldiagnosis?

A. Exacerbation of chronic pulpitisB. Acute periodontitisC. Exacerbation of chronic periodontitisD. Acute serous pulpitisE. Acute suppurative pulpitis

25. A 10-year-old girl complains of sensations ofdryness and pain in her lips, which develop inthe summer. On examination: the red borderhas areas of congestive hyperemia and infiltrati-on, scales and scabs, that, when removed, resultin hemorrhaging erosions. The skin surroundinglips is unaltered. No rashes are detected on theoral mucosa. What is the most likely diagnosis?

A. Actinic cheilitis, dry formB. Actinic cheilitis, exudative formC. Atopic cheilitisD. Allergic contact cheilitisE. Meteorological cheilitis

26. An 8-year-old girl complains of tooth 21 di-scoloration and pain response to hot stimulus.Several months ago tooth 21 was treated foracute diffuse pulpitis by means of vital amputati-on. Objectively tooth 21 is filled, percussion ispainless. X-ray shows the rooth to be formed by2/3, cortical plate of the tooth socket remainsintact in the area of root radix. What materialshould be used for root canal filling in this case?

A. Calcium hydroxide pasteB. Zinc phosphate cementC. Resorcinol-formalin pasteD. Glass ionomer cementE. -

27. A 10-year-old child complains of persisti-ng throbbing pain in tooth 36, which appearedone day ago. Hot stimulus aggravates thepain, while cold mitigates it slightly. Objecti-vely on the masticatory and medial surfacesof tooth 36 there is a deep carious cavity non-communicating with the dental cavity. Probingof the cavity bottom and percussion are pai-nful. X-ray shows no pathologic changes of theperiodontium. What treatment method shold bechosen in this case?

A. Vital extirpationB. Devital extirpationC. Vital amputationD. Devital amputationE. Conservative treatment

28. A 8,5-year-old child is mostly healthy. Thereis a complaint of pain in the upper left tooth,due to it having been physically damaged 3hours ago. Objectively: 1/2 of the 21 tooth crownis destroyed, the pulp is significantly exposed,red, sharply painful and bleeding when probed.Percussion of the 21 tooth is sharply painful.Choose the optimal method of the 21 toothtreatment:

Page 29: Krok 2 - testcentr.org.ua · D. Monoblastic leukemia E. Acute herpetic stomatitis 18. A 57-year-old retired man complains of attacks of burning pain and rashes on the skin of his

Дитяча терапевтична стоматологiя 29

A. Vital amputationB. Vital extirpationC. Devital amputationD. Devital extirpationE. Biological method

29. A 11-year-old child complains of pain in thelower right lateral tooth, which occurs wheneating, especially hot food. On the masticatorysurface of the 46 tooth there is a large cariouscavity filled with softened light-brown dentin.The cavity is located within parapulpar dentin.In the projection of the medial buccal pulp hornthe carious cavity communicates with the pulpchamber. Deep probing is painful. Electric pulptest - 60 microamperes. Make the diagnosis:

A. Chronic gangrenous pulpitisB. Chronic hypertrophic pulpitisC. Acute diffuse pulpitisD. Chronic fibrous pulpitisE. Acute focal pulpitis

30. A 7-year-old practically healthy child wasundergoing the carious cavity preparation oftooth 46 due to acute median caries. Duringthis procedure the mesio-buccal pulp horn wasaccidentally exposed. What treatment would beoptimal in this case?

A. Biological approachB. Devital amputationC. Devital extirpationD. Vital amputationE. Vital extirpation

31. A 9-year-old girl complains of persistingpain in tooth 11 that one month ago sustai-ned a trauma resulting in broken crown. Thetooth received no treatment. Objectively the1/4 of the tooth crown of 11 is broken off, theremaining crown is grayish, the dental cavity isnot exposed. Percussion is acutely painful. Themucogingival fold is hyperemic and acutely pai-nful on palpation. Make the diagnosis:

A. Acute traumatic periodontitisB. Acute diffuse suppurative pulpitisC. Acute suppurative periodontitisD. Exacerbation of chronic periodontitisE. Chronic granulating periodontitis

32. A 10-year-old boy complains of painful sorein the mouth, which has been persisting andincreasing in size for 1,5 months. Objectively: onthe buccal mucosa there is a soft shallow ulcer 2cm in diameter with uneven undermined edges.The floor of the ulcer is tuberous, covered inyellow-gray coating. The ulcer is surrounded wi-th numerous yellowish tubercles. The regionallymph nodes are elastic, painful, and mattedtogether. These symptoms are characteristic ofthe following disease:

A. TuberculosisB. Lichen ruber planusC. Necrotizing ulcerative stomatitisD. CancerE. Syphilis

33. A 4-year-old child has developed acute

spontaneous pain in the tooth on the lower rightjaw, which aggravates on biting. Objectively: inthe 85 tooth there is a deep carious cavity non-communicating with the dental cavity. Probi-ng is sharply painful at all points of the cavityfloor. Painful reaction to cold water stimulus andpercussion is observed; mucosa surrounding the85 is hyperemic. Submandibular lymphadenitisis detected. Make the provisional diagnosis:

A. Acute pulpitis complicated with periodontitisB. Acute serous periostitisC. Acute serous periodontitisD. Acute suppurative pulpitisE. Exacerbation of chronic periodontitis

34. A 14-year-old boy complains of rapidwearing-off of tooth crowns. Objectively: toothcrowns are worn-off by 1/3. Enamel easily chipsoff and is pale gray in color. Make the diagnosis:

A. Stainton-Capdepont syndromeB. Dentinogenesis imperfectaC. FluorosisD. Systemic hypoplasiaE. Focal hypoplasia

35. During the oral cavity sanation on the vesti-bular surface of the 21 and 12 teeth in thecervical area there were detected chalky spots.Enamel surface is dull and coarse, no reacti-on to thermal stimuli. Decayed-Missing-FilledIndex (dmft/DMFT) is 6, Hygiene Index is 2.Mesial occlusion is observed. Within the firstyear of life the patient had been suffering fromfrequent cases of URTI and a case of chickenpox. What additional investigation methodswould be useful for the diagnosis-making in thiscase?

A. Vital stainingB. Anamnesis dataC. Electric pulp testD. X-rayE. Stomatoscopy

36. Objective examination of a 10-year-old chi-ld revealed slight hyperemia, infiltration, anddryness of the whole surface of the vermillionborder. Architectonics of the lips is disturbed.Dryness and contracted sensation are observedin the lips, especially during cold seasons. Makethe provisional diagnosis:

A. Meteorological cheilitisB. Atopic cheilitisC. Allergic contact cheilitisD. Exfoliative cheilitisE. Cheilitis of microbial origin

37. A 14-year-old child complains of throbbingundulating pain in the lower left teeth, whichaggravates due to hot stimuli. Objectively: onthe masticatory surface of the 36 tooth there is acarious cavity within parapulpar dentin, which isnon-communicating with the dental cavity. Thecavity floor probing is painless, tooth percussi-on is painful. What treatment method would beoptimal in the given case?

Page 30: Krok 2 - testcentr.org.ua · D. Monoblastic leukemia E. Acute herpetic stomatitis 18. A 57-year-old retired man complains of attacks of burning pain and rashes on the skin of his

Дитяча терапевтична стоматологiя 30

A. Vital extirpationB. Devital extirpationC. Devital amputationD. Vital amputationE. Biological method

38. A 2.5-year-old child has fever up to 38.5oC ,low appetite, rashes in the oral cavity. The di-sease onset was 3 days ago. Objectively: the skinof the perioral area is covered in scarce vesicleswith clear content. Within the oral cavity on thebuccal and lingual mucosa there are sharply pai-nful erosions, 2-3 mm in size, with white coatingand hyperemic crown. The gums are swollen,hyperemic. The submandibular lymph nodesare enlarged, painful on palpation. Make thediagnosis:

A. Acute herpetic stomatitisB. Stevens-Johnson syndromeC. Erythema multiforme exudativumD. Stomatitis with the background of infectiousmononucleosisE. Stomatitis with the background of chickenpox

39. A 10-year-old boy complains of acute painattacks in the area of his upper left teeth.The toothache persisted for a night. Objecti-ve examination revealed a carious cavity onthe masticatory surface of the 26 tooth withinparapulpar dentin. Probing is sharply painful atall points of the cavity floor. Markedly positi-ve reaction to cold water stimulus is observed.Select the most likely diagnosis:

A. Acute diffuse pulpitisB. Acute serous periodontitisC. Acute suppurative pulpitisD. Acute suppurative periodontitisE. Acute local pulpitis

40. A 15-year-old patient complains of cari-ous cavity and short-term ”lightning-fast” painattacks in the 26 tooth. The pain attacks cease in1-2 minutes after eating. Objectively: there is adeep carious cavity filled with softened dentin.The cavity floor is painful on probing. Make thediagnosis:

A. Pulpal hyperemiaB. Acute traumatic pulpitisC. Acute suppurative pulpitisD. Acute local pulpitisE. Acute diffuse pulpitis

41. Mother of a 10-year-old girl complainsof a cosmetic defect of the child’s 22 tooththat erupted with damaged enamel. Anamnesisstates premature extraction of the 62 tooth dueto caries complication. There is a white-yellowspot with clear margins on the vestibular surfaceof the 22 tooth. Enamel retains glossiness, no

surface roughness can be detected on probing.Make the diagnosis:

A. Local enamel hypoplasiaB. FluorosisC. Acute superficial cariesD. Chronic superficial cariesE. Systemic enamel hypoplasia

42. A 16-year-old adolescent girl complains ofpain caused by cold stimuli and food particlesretained in her upper jaw tooth. Objectively:on the contact surface of the 24 tooth there isa carious cavity within parapulpar dentin. Thecavity floor and walls are covered with lightsoftened dentin. The carious cavity floor is sensi-tive to probing, percussion of the 24 is painless.Cold water stimulus is painful, the pain quicklyabates after the stimulus is removed. Make thediagnosis:

A. Acute deep cariesB. Acute median cariesC. Acute diffuse pulpitisD. Chronic fibrous pulpitisE. Chronic deep caries

43. A 15-year-old girl complains of brief painattacks in her teeth due to chemical stimuli.Objectively: on the contact surfaces of the 11,21, and 22 teeth there are enamel areas mattwhite in color, with lost shine, covered in largeamount of dental deposit. Enamel is softenedand can be easily chipped off with excavator.Probing of lesions is painless. Percussion is pai-nless. No reaction to cold stimuli. Make the di-agnosis:

A. Acute superficial cariesB. Acute median cariesC. Acute initial cariesD. Chronic initial cariesE. Chronic superficial caries

44. A 5.5-year-old child is undergoing preventi-ve examination. There are no complaints fromthe patient. Objectively: in the precervical areaof buccal surface of the 75 tooth there is anenamel patch with loss of natural glossiness. Theenamel surface is coarse, painless on probing.Tooth percussion is painless. Thermodiagnosisis negative. Damaged enamel stains with 2%water solution of methylene blue. Make the di-agnosis:

A. Acute initial cariesB. Enamel hypoplasiaC. FluorosisD. Acute superficial cariesE. Chronic initial caries

Page 31: Krok 2 - testcentr.org.ua · D. Monoblastic leukemia E. Acute herpetic stomatitis 18. A 57-year-old retired man complains of attacks of burning pain and rashes on the skin of his

Дитяча хiрургiчна стоматологiя 31

1. A 10-year-old child is referred by theorthodontist for extraction of tooth 53. Objecti-vely the crown of 53 is retained, the toothis immobile. X-ray of tooth 53 shows rootresorption by less than 1/3. Choose the bestinstrument for extraction of tooth 53:

A. Straight crown forcepsB. Straight elevatorC. Root bayonet forcepsD. Crown forceps with S-shaped handlesE. Beak-shaped root forceps

2. A 7-year-old boy is diagnosed with epi-demic parotitis (mumps). Name the most likelycomplication of this disease:

A. OrchitisB. ColitisC. DermatitisD. PneumoniaE. Cholecystitis

3. A 4-year-old boy has been diagnosed withacute purulent periostitis of the upper jaw ori-ginating from the 64 tooth. Choose the optimaltreatment tactics:

A. The 64 tooth extraction, periosteotomy,pharmacotherapyB. The 64 tooth extraction, anti-inflammatorypharmacotherapyC. Endodontological treatment of the 64 tooth,anti-inflammatory pharmacotherapyD. Endodontological treatment of the 64 tooth,periosteotomyE. Periosteotomy, anti-inflammatorypharmacotherapy

4. An adolescent complains of reduced andpainful mouth opening, difficulties when eati-ng, and swelling in the left mandibular anglethat developed after tooth 37 was extracted 3days ago. Objectively the face is asymmetricdue to soft tissue swelling in the area of theleft mandibular angle. Mouth opening is pai-nful and reduced to 2.0 cm. Disturbed occlusi-on is observed. Palpation of the left mandi-bular angle is painful, the tissues are soft, bonecrepitus is detected. ”Indirect load to the chi-n” symptom is positive in the area of the leftmandibular angle. The socket of the extractedtooth is packed with iodoform gauze. What isthe most likely diagnosis?

A. Left mandibular angle fractureB. Mandibular alveolar fractureC. Anterior mandibular fractureD. Mandibular periostitis on the leftE. Odontogenic mandibular osteomyelitis

5. Puncture sample taken from a 13-year-oldchild contains giant Reed-Sternberg cells. Whatdiagnosis can be confirmed by the cell contentof this puncture material?

A. LymphogranulomatosisB. Tuberculous lymphadenitisC. Lymphocytic leukemiaD. Lymph node actinomycosisE. Infectious mononucleosis

6. A 7-year-old child complains of pain andswelling in the left submandibular region. Theswelling in this region developed 2 days ago.Objectively: the child is in a satisfactory condi-tion, body temperature is of 37.3oC . Face isasymmetrical due to the soft tissue swellingin the left submandibular region. Palpationreveals a round formation 2x2 cm in size. Theformation is mobile, painful, non-fused withthe skin. The 74 tooth is discolored, percussionis painful. What is the provisional diagnosis?

A. Acute serous odontogenic lymphadenitis ofthe left submandibular regionB. Acute serous nonodontogenic lymphadenitisof the left submandibular regionC. Acute suppurative odontogenic lymphadeni-tis of the left submandibular regionD. Phlegmonous adenitis of the right submandi-bular regionE. Lateral cervical cyst

7. A 13-year-old boy complains of generalweakness, high body temperature up to 39oC ,lack of appetite, constant pain in the bodyof the lower jaw. Objectively: observed issignificant asymmetry of the face causedby soft tissues swelling in the left buccaland submandibular areas. Mouth opening isreduced. Intraoral examination revealed thefollowing: the 34, 35, 36, and 37 teeth are mobi-le; teeth percussion is painful. The crown of the36 tooth is completely destroyed. The mucosaof those teeth is hyperemic and painful whenpalpated. Muff-like enlargement of the lowerjaw alveolar process is detected. What is themost likely diagnosis?

A. Acute mandibular odontogenic osteomyelitisB. Acute mandibular hematogenousosteomyelitisC. Acute mandibular odontogenic suppurativeperiostitisD. Ewing’s sarcomaE. Abscess of the right submandibular area

8. A 3-year-old child received an injury of theupper teeth as a result of a fall. Objectively:crowns of the 51 and 61 teeth are embeddeddeep into the surrounding tissues with onlytheir cutting edge visible, the gingival marginis hyperemic, edematic. What is the treatmenttactics?

A. Tooth extractionB. MonitoringC. RepositionD. Endodontic treatmentE. Anti-inflammatory therapy

9. An 8-year-old child has been clinically di-agnosed with exacerbation of chronic peri-odontitis of the 84 tooth. The crown is decayedby 1/2. What is the optimal tactics of dental

Page 32: Krok 2 - testcentr.org.ua · D. Monoblastic leukemia E. Acute herpetic stomatitis 18. A 57-year-old retired man complains of attacks of burning pain and rashes on the skin of his

Дитяча хiрургiчна стоматологiя 32

treatment?

A. ExtractionB. Endodontic treatmentC. Endodontic treatment and drug therapyD. Drug therapyE. Opening along the mucogingival fold, drugthrapy

10. Parents of a 3-year-old child complain offood periodically getting into the child’s nasalcavity during feeding. Objectively there is a fi-ssure in the area of the soft palate. Make thediagnosis:

A. Isolated partial nonunion of the soft palateB. Congenital hidden cleft palateC. Isolated nonunion of the hard and soft palateD. Cleft hard palateE. Combined cleft palate

11. Due to trauma of the area of teeth 44and 45, a 12-year-old boy suffers from thepathologic displacement of the mandibularalveolar process and rupture of the alveolarprocess mucosa. What additional examinationis necessary to specify the diagnosis?

A. X-ray of the mandible in frontal and lateralprojectionsB. Skull X-ray in axillary projectionC. X-ray of the mandible in frontal and ParmaprojectionD. Computed tomography of the mandibleE. -

12. A 12-year-old child presents withtemperature 38oC , chills, nausea, vomiti-ng, delirium, and weakness. On the mi-ddle third of the face there is hyperemia ina butterfly-shaped pattern. Regional lymphnodes are enlarged and mildly painful. Inblood: leukocytes - 12 · 109/L, lymphocytes -8.0 · 109/L, ESR- 26 mm/hour. What diagnosisshould be made?

A. Erysipelatous inflammationB. Facial vein thrombophlebitisC. Cutaneous actinomycosisD. Streptoderma, submandibularlymphadenopathyE. Acute non-odontogenic maxillary sinusitis

13. A 7-year-old boy came to a dental surgeonwith complaints of painful swelling of his ri-ght cheek and high body temperature. Objecti-vely the body temperature is 38,2oC , facialasymmetry caused by the right cheek edema,no skin discoloration, skin can be pinched ina fold, no mouth opening reduction. In theoral cavity the mucogingival fold in the areaof the 84 and 85 teeth is smoothed out, thefold is hyperemic, fluctuation is observed. The85 tooth is destroyed by cariosity, painless onpercussion. Make the diagnosis:

A. Acute odontogenic suppurative mandibularperiostitis originating from the 85 toothB. Acute odontogenic albuminous mandibularperiostitis originating from the 85 toothC. Acute odontogenic mandibular osteomyelitisoriginating from the 85 toothD. Suppuration of the periapical cyst of the 85toothE. Odontogenic abscess of the buccal areaoriginating from the 85 tooth

14. A 15-year-old adolescent boy complains ofpain in the lower right jaw, which increasesduring chewing, and impaired closure of theteeth. Anamnesis: 2 days ago a trauma wasreceived. Based on objective examinationresults and X-ray data the patient was di-agnosed with open fracture of the mandi-ble between the 45 and 46 teeth. Choose themethod of treatment:

A. Removable bracesB. Temporary immobilizationC. Port’s dental splintD. Rudko’s applianceE. Ligature

15. Parents complain of painfulness and mobi-lity of the tooth of their 4-year-old child, whi-ch developed after the impact with a woodenobject. Objectively: the face is asymmetricaldue to swollen tissues of the upper lip. The51 tooth is intact, with vestibular displacementand the II degree mobility, gums around the51 tooth are hyperemic. What provisional di-agnosis can be made?

A. Incomplete dislocation of the 51 toothB. Complete dislocation of the 51 toothC. Contusion of the 51 toothD. Acute albuminous periostitisE. Acute suppurative periostitis

16. A 7-year-old child is in a grave conditi-on, teeth of the upper left jaw are painful,body temperature is 39, 3oC . The child is pale,adynamic; the face is asymmetrical due to infi-ltration in the upper left jaw. The 64 tooth is fi-lled, painful on percussion. The 63 and 65 teethare intact, painful on percussion. The I degreeof tooth mobility is observed; pus is being di-scharged from under the marginal gingiva ofthe 64 tooth. The alveolar process is deformedat its vestibular and palatine surfaces. Make theprovisional diagnosis:

A. Acute odontogenic osteomyelitisB. Acute albuminous periostitisC. Acute suppurative periostitisD. Ossification periostitisE. Ewing’s sarcoma

17. A child is diagnosed with congenital cleft inthe soft palate and posterior part of the hardpalate. What type of anesthesia should be gi-ven to the patient for uranostaphyloplasty?

Page 33: Krok 2 - testcentr.org.ua · D. Monoblastic leukemia E. Acute herpetic stomatitis 18. A 57-year-old retired man complains of attacks of burning pain and rashes on the skin of his

Дитяча хiрургiчна стоматологiя 33

A. Intubation narcosisB. Intravenous narcosisC. Anesthesia maskD. Infiltration anesthesiaE. Conduction anesthesia

18. A 6-year-old child complains of pain andedema in the upper right jaw, body temperatureup to 37.9oC , and deterioration of general well-being. Symptom onset was 3 days ago. Objecti-vely the face is asymmetric due to soft tissueedema of buccal and infraorbital regions onthe right. The crown of 54 is destroyed by1/2, percussion is painful; the tooth previouslyhad been treated for complicated caries. Onthe palatine side of the affected tooth areathere is a painful infiltration with fluctuationin its center; the tissues over the infiltration arehyperemic. Make the provisional diagnosis:

A. Acute suppurative periostitis of the maxillaoriginating from tooth 54B. Acute serous periostitis of the maxilla origi-nating from tooth 54C. Acute odontogenic osteomyelitis of themaxillaD. Exacerbation of chronic periodontitis of 54E. Chronic odontogenic osteomyelitis of themaxilla

19. An ambulance has delivered an 8-year-oldchild to an admission room. An oral surgeonhas made the following diagnosis: odontogenicphlegmon of the right submandibular area.What surgical approach would be advicable forsurgical treatment of this phlegmon?

A. Dissection in the submandibular area,parallel to the mandibleB. Dissection parallel to the torus mandibularisC. Dissection around the mandibular angleD. Dissection along the lower neck foldE. Dissection in the area of pterygomandibularfold

20. Mother of an 8-month-old girl came to aclinic with complaints of the child’s anxiety,fussiness, high fever up to 38, 5oC , signs of ali-mentary canal irritation, vomiting and refusalto eat. On objective examination the child ispale, crying, presents with hyperemia, edema,gingival pain in the frontal area of the upperjaw, no erupted teeth can be detected. Make

the diagnosis:

A. Hindered tooth eruptionB. Acute herpetic stomatitisC. Food poisoningD. Hematogenous osteomyelitis of the maxillaE. Odontogenic osteomyelitis of the maxilla

21. A 7-year-old girl hit her forehead one dayago. Several hours after the sustained trauma aswelling developed in the left superciliary area.General condition of the child is unaffected.Objectively: swelling of the forehead tissuesspreading towards the left eyelids; the swelli-ng is soft, fluctuation sign is present. Make thepreliminary diagnosis:

A. Hematoma of the left superciliary areaB. Hemangioma of the right superciliary areaC. Fracture of the temporal boneD. Fracture of the frontal boneE. Hematic abscess of the left superciliary area

22. Parents of a 3-year-old child complain thatthe child has a neck growth that developed3 months after the birth. Objectively: in theupper lateral neck area there is a semici-rcular neoplasm with limited mobility, softelastic consistency, no skin alterations, pai-nless on palpation. Puncture yielded pus-likeclear yellow substance. Make the provisionaldiagnosis:

A. Branchial cleft cystB. Chronic lymphadenitisC. LymphangiomaD. Specific lymphadenitisE. Hemangioma

23. Parents of a 6-year-old child complainof their child having a gradually enlargingneoplasm in the left parotid-masticatory regi-on. Skin over the tumor is without discolorati-on. The tumor is painless, but when the headbends down the tumor increases in size andassumes bluish coloring. What disease can besuspected in the child?

A. HemangiomaB. FibromaC. AtheromaD. LymphangiomaE. Cyst of the parotid gland

Page 34: Krok 2 - testcentr.org.ua · D. Monoblastic leukemia E. Acute herpetic stomatitis 18. A 57-year-old retired man complains of attacks of burning pain and rashes on the skin of his

Ортодонтiя 34

1. A girl is 8 years old. She complains ofimpaired mastication. Objectively: on examinati-on of the oral cavity the cutting edges of herlower incisors touch the palatine mucosa in thefrontal area; the upper frontal teeth overlap wi-th the lower ones by full height of their crowns.On the lower jaw the occlusal curve of the frontteeth is markedly concave. Make the provisionaldiagnosis:

A. Deep overbiteB. Open biteC. False prognathismD. True prognathismE. Cross bite

2. A boy is 10 years old. He complains of slopedchin and impaired mastication. Anamnesis statesformula feeding. Objectively: corellation of the6th teeth is of the Angle’s II class. Sagittal fissureis 7 mm. Eschler-Bittner test is positive. What isthe most likely diagnosis?

A. Prognathism, distal mandibular displacementB. Prognathism, maxillary macrognathiaC. Retrusion of the lower jaw frontal areaD. Progenia, mandibular macrognathiaE. Protrusion of the upper jaw frontal area

3. After adenotonsillectomia it is necessary tobreak the mouth breathing habit in a 4-year-oldchild. The orthodontist recommends applicationof an oral vestibular shield (Kerbitz’ vestibularplate). Vestibular shield facilitates training of thefollowing muscle:

A. Orbicular muscleB. Temporal muscleC. Masseter muscleD. Lateral pterygoid muscleE. Medial pterygoid muscle

4. What denture constructions should be chosenin the cases of multiple adentia during the initialperiod of occlusion change?

A. Removable partial dentureB. Dental bridgeC. Clasp-retained (bugel) removable partialdentureD. No denture is necessaryE. Removable complete denture

5. How often should the dentures be replacedin children during the period of milk occlusionaccording to Ilyina-Markosian?

A. Every 6-8 monthsB. Every 8-10 monthsC. Every 10-12 monthsD. Every 12-16 monthsE. Every 16 months

6. A child is 8 years old. There are complaintsof congested upper incisors. Objectively: the fi-rst molars closure is of Angle’s I class, frontaloverbite is orthognathic. The 12 and 22 teetherupt palatinally with space deficiency of 2/3of the tooth crown. The 11 and 21 teeth are 10mm each in cross-section. The child has inheri-ted father’s facial type with prognathism andmacrodontia of the central incisors. Choose thepreventive treatment, considering this hereditary

pathology:

A. Hotz serial extraction to reduce the dentalarchB. Jaw expansion to provide the space for the 12and 21 teethC. Massage of the 12 and 21 teeth area to sti-mulate their eruptionD. Extraction of the 12 and 21 teeth to reduce thedental archE. Shave off the approximal surfaces of the 11and 21 to provide the space for the 12 and 22 teeth

7. A 5-year-old child has bad habit of suckingon his tongue. At the front area there is a smallvertical fissure up to 2 mm in size. Neutral closureis observed in the lateral areas of the jaws. Thechild is diagnosed with open traumatic bite of theI degree. A vestibulo-buccal shield was prescri-bed for treatment. What is the function of theappliance in the given case?

A. Treatment and preventionB. PreventionC. RetentionD. PassiveE. Treatment

8. Parents of an 8-year-old child have made andappointment with an orthodontist. There arecomplaints of their child having traumas of oralmucosa. Objectively: decreased height of thelower face, everted lower lip, deep labiomentalfurrow, milk occlusion. The upper incisors fullycover the lower ones; cutting surface of the lowerincisors make contact with the anterior third ofthe palate. Mesiodistal ratio of the canines andthe first permanent molars is normal. Groupingof the upper and lower front teeth is dissimi-lar. Make the diagnosis according to the Kalvelisclassification:

A. Deep traumatic overbiteB. Deep incisor overbiteC. Deep neutral occlusionD. Deep prognatic (roof-shaped) occlusionE. -

9. Preventive examination of a 5-year-old chi-ld revealed a habit of lower lip biting. Whatmalocclusion may develop if the child keeps thishabit?

A. Anterior biteB. Prognathic biteC. Open biteD. Deep overbiteE. Cross-bite

10. A 7-year-old child has protruding chin, thelower lip overlaps the upper one. There are di-astema and tremata between the lower incisors,the lower incisors overlap the upper incisors by2/3 of the crown height. First permanent molarsdemonstrate Angle’s class III relation. Sagittalgap is 3 mm. The correct treatment tactics wouldbe to:

Page 35: Krok 2 - testcentr.org.ua · D. Monoblastic leukemia E. Acute herpetic stomatitis 18. A 57-year-old retired man complains of attacks of burning pain and rashes on the skin of his

Ортодонтiя 35

A. Use Bruckl’s applianceB. Recommend a complex of myogymnasticexercisesC. Use Angle’s slider applianceD. Use Bynin’s applianceE. Use Schwartz’s appliance

11. During preventive examination a 5-year-old child was determined to have insufficientphysiological attrition of the cusps of the deci-duous canines. What treatment tactics shouldthe doctor choose?

A. To shave off the retained canine cuspsB. Medical examination once a month until theincisors are replacedC. Medical examination every 6 months until theincisors are replacedD. Medical examination every 6 months until thecanines are replacedE. No medical intervention is necessary

12. During examination of a 5-year-old childthe orthodontist revealed no wear of teeth, notremata and diastemata, orthogenic occlusion.Which of the following symptoms detected inthe 5-year old child is a sign of future teethovercrowding?

A. Absence of tremata and diastemataB. Absence of wear of teethC. Orthogenic occlusionD. Orthognathic biteE. Absence of mesial step in the region of secondtemporary molars

13. Teeth 71 and 81 erupted in a 6-year-old child,the lower jaw is retrogenic, the palate is flat withpronounced cross-folds. Determine the conditi-on of the oral cavity:

A. PhysiologicB. PathologicC. AbnormalD. SubcompensatedE. Decompensated

14. A 2.5-year-old child is registered forregular check-ups with the orthodontist. TheI stage of physiogical occlusion developmentcorresponds with eruption of the following groupof temporary teeth:

A. Temporary molarsB. Temporary central incisorsC. Temporary lateral incisorsD. Temporary caninesE. Front teeth

15. An orthodontist has been addressed byparents of a 5-year-old child. The child has the54 tooth extracted, all the other deciduous teethare present. The doctor made a thin-wall crownfor the 55 tooth with interdental wedge to the 53tooth. What is the purpose of such treatment?

A. Prevention of dentition malformationB. Aesthetic restorationC. Restoration of masticatory efficiencyD. Acceleration of permanent tooth eruptionE. Deceleration of permanent tooth eruption

16. During Eschler-Bittner test the profile ofa 12-year-old girl with posterior occlusion hasshown some improvement. Specify the conditi-on that resulted in the development of posteriorocclusion in this patient:

A. Mandibular underdevelopmentB. Maxillary overdevelopmentC. Mandibular underdevelopment and maxillaryoverdevelopmentD. Mandibular overdevelopmentE. Maxillary underdevelopment

17. Parents of an 8-year-old girl complain of theirchild having an aesthetic defect of her teeth.Objectively the patient’s lower face is shortened.Her chin protrudes forwards and her upper lip issunken. During teeth closure the deep underbitebecomes apparent. Mesio-occlusion is observedin the lateral areas. Choose the apparatus opti-mal for the treatment:

A. Frankel functional regulator - 3B. Frankel functional regulator - 2C. Osadchy apparatusD. Andresen-Haupl activatorE. Frankel functional regulator - 1

18. Parents of a 7-year-old child addressed ahospital with complaints of their child having nopermanent teeth in the front area of the mandi-ble. Anamnesis states that the first deciduousteeth erupted at the age of 11 months. Objecti-ve clinical examination revealed the following:appearance is without changes; milk occlusion;there are physiological diastema and tremata;edge-to-edge incisor contact. What preliminarydiagnosis can be made according to Kalvelisclassification?

A. Retarded eruptionB. Supernumerary toothC. AdentiaD. DystopiaE. Hypoplasia

Page 36: Krok 2 - testcentr.org.ua · D. Monoblastic leukemia E. Acute herpetic stomatitis 18. A 57-year-old retired man complains of attacks of burning pain and rashes on the skin of his

Ортодонтiя 36

1. A patient needs his 26 tooth extracted.After application of tuberal anaesthesia hedeveloped general fatigue, nausea, and, later,severe itching and skin rashes. What complicationoccurred in the patient?

A. UrticariaB. Anaphylactic shockC. CollapseD. Quincke’s edemaE. Unconsciousness

2. Prior to dental treatment a 13-year-old patienthad been administered anaesthesia. The patientcomplained of itching, tingling skin of the face,vertigo, nausea, labored respiration, spontaneousvision impairment. Objectively: pale face, swolleneyelids and red border, dilated pupils, threadypulse, and rapid labored respiration with crackles.Make the diagnosis:

A. Anaphylactic shockB. SyncopeC. CollapseD. Epileptic attackE. Quincke’s edema

3. A 42-year-old woman has made an appoi-ntment with a prosthodontic office to make adenture. Objectively: dental formula is as follows:

18 . . . . 13 12 11 21 22 23 . . . . 2848 47 46 45 44 43 42 41 31 32 33 34 35 36 37 .

The patient has deep occlusion; clinical crownsare low; equator is not pronounced. The pati-ent suffers from epileptic seizures. What kind ofdenture should be prescribed for this patient?

A. Removable partial laminar metal-baseddentureB. Dental bridgeC. Removable partial laminar plastic denture withretainers (clammers)D. Removable partial laminar denture withsupporting-retaining clasps (clammers)E. Clasp-retained (bugel) removable partialdenture

4. A 56-year-old patient suffering fromexacerbation of schizophrenia has been hospi-talized in an oral in-patient department with adiagnosis of the displaced mandibular fracturein the area of teeth 34 and 35. What method oftreatment should be prescribed?

A. OsteosynthesisB. One arch smooth dental bracesC. Full dental splintD. Weber splintE. Vankevych splint

5. A 25-year-old HIV-infected patient cameto a clinic of prosthetic dentistry to have adenture made for him. What aseptic and anti-septic precautions should be taken?

A. According to the schemeB. The impressions should be desinfected under aquartz lampC. The patient should be refused appointmentD. No special precautions are requiredE. The orthopaedist should work in gloves and amask

6. A 7-year-old child is diagnosed with chronicgranulating periodontitis of the 55 tooth. Addi-tionally accompanying diagnosis of rheumaticendocarditis is made. What treatment tacticsshould a dentist choose?

A. Tooth extractionB. Endodontic treatmentC. Endodontic treatment and physical therapyD. Case monitoringE. Endodontic treatment and case monitoring

7. A 6-year-old boy with congenital heart di-sease (pulmonary artery stenosis) presents withsuppurative periostitis of the maxilla. The childneeds surgical treatment. What unit should he bereferred to?

A. The pediatric maxillofacial unitB. The out-patient unit, no precautions arenecessaryC. The out-patient unit after preliminary cardi-ological treatmentD. The cardiology unitE. The out-patient or in-patient unit at the di-scretion of the child’s parents

8. A patient with ischemic heart disease hasdeveloped ventricular fibrillation. What is thefirst-priority therapeutic action?

A. Electric defibrillationB. Lidocaine injectionC. Adrenaline injectionD. Potassium chloride injectionE. Novocaine amide injection

9. A patient needs the 36 tooth extracted.After administering anesthesia the doctor startedapplying the elevator. However, immediatelyafter that the patient suddenly paled, complai-ned of dizziness, ear noise, and blackout and sliddown in the chair. What is the most likely di-agnosis?

A. UnconsciousnessB. Anaphylactic shockC. CollapseD. ShockE. Hypoglycemic coma

10. A 34-year-old man came to a dental clinic forextraction of the 26 tooth. After application of1,7 ml of Ultracain (Articaine) solution for localanaesthesia the patient developed general fatigueand nausea. Objectively: the skin is pale, cold,cyanotic, covered in clammy sweat; BP is 60/40mm Hg. What urgent condition did the patientdevelop?

A. CollapseB. Anaphylactic shockC. Loss of consciousnessD. Bronchial asthmaE. Urticaria

11. A 68-year-old patient addressed a surgicaldepartment of a dental clinic for extraction of the45 tooth. During procedure the patient developedburning retrosternal pain attack irradiating to theleft shoulder, scapula, hand. The skin is pale,BP is 140/100 mm Hg, heart rate is rapid. Skinhyperplasia can be observed in the Zakharin-Head’s zones. What emergency condition did the

Page 37: Krok 2 - testcentr.org.ua · D. Monoblastic leukemia E. Acute herpetic stomatitis 18. A 57-year-old retired man complains of attacks of burning pain and rashes on the skin of his

Ортодонтiя 37

patient develop?

A. Angina pectoris attackB. Heart failureC. Bronchial asthma attackD. Hypertensic crisisE. -

12. A 7-year-old girl received conductionanesthesia with 2% articaine solution for extracti-on of tooth 16. She has no history of allergies.After receiving anesthesia the patient complai-ned of weakness, she developed skin pallor,cyanosis, and nausea. Her blood pressuredropped significantly. The patient is conscious.What is the provisional diagnosis?

A. CollapseB. Anaphylactic shockC. Allergic response to the anestheticD. Pain shockE. Vertigo

13. A 65-year-old man during the tooth extracti-on suddenly felt unwell, he developed severe painirradiating to the left scapula and numbness of theleft hand. Objectively: the patient is pale, beadsof perspiration appeared on his forehead, BP is170/90 mm Hg, heart rate is 86/min., rhythmical.The dentist stopped the manipulations in the oralcavity. What drug should be administered in thiscase?

A. NitroglycerineB. Zelenin dropsC. Valerian tinctureD. ValocordinE. Analgin (Metamizole)

14. A 45-year-old patient after administration oflocal anesthesia in preparation for oral surgeryhas suddenly felt unwell, developed increasingedema of laryngeal mucosa and respiration di-sorder. The dentist stopped the manipulations inthe oral cavity. What type of asphyxia developedin the patient?

A. StenoticB. DislocationalC. ValvularD. ObturativeE. Aspiration

15. A 19-year-old young man, who was waitingfor the appointed time at the dentist’s, suddenlydeveloped an attack: his face became purple,bloody foam flowed from his mouth (bittentongue), pupils were dilated and unresponsiveto light, the patient developed first tonic thenclonic convulsions that stopped spontaneously,after that he calmed down and fell asleep quickly.What happened with the patient?

A. Epileptic seizureB. Sympathoadrenal crisisC. SpasmophiliaD. Morgagni-Adams-Stokes syndromeE. Pulmonary embolism

16. A victim of a traffic accident was deliveredinto the admission room. The patient is supineand unconscious. His skin is cyanotic, respirationis extremely labored, mucosa is pale, blood clotsare accumulated in the oral cavity. The patientis diagnosed with displaced bilateral mandibularfracture. How should this patient be transported?

A. In the prone position on a soft stretcherB. Positioned on the side on a rigid stretcherC. In the sitting position with the head thrownbackD. In the sitting position with air tube insertedinto the upper airwaysE. Positioned on the side on a soft stretcher

17. A victim of a traffic accident was deliveredinto the admission room. The patient is supineand unconscious. His skin is cyanotic, respirati-on is extremely labored, mucosa is pale, bloodclots are accumulated in the oral cavity. The pati-ent is diagnosed with displaced bilateral mandi-bular fracture. What measures should be taken toprevent complications in this case?

A. Fixation of the lower jaw and tongue withstandard Entin’s head-chin strapB. Fixation of the tongue to the patient’s collarC. Tracheostomy and artificial pulmonary venti-lationD. Removal of foreign bodies from the oral cavityE. Excision of injured mucosal flaps

18. A 5-year-old child developed a hemorrhageafter pulp extirpation of 74 due to exacerbatedchronic pulpitis. The child suffers from Von Wi-llebrand disease. What actions should be takenby the dental surgeon to stop bleeding?

A. To hospitalize the child to the hematologicalunitB. To hospitalize the child to the maxillofacial unitC. To place sutures on the mucosaD. To plug the cavity with epsilon-aminocaproicacid dressingE. To plug the cavity with hemostatic sponge

19. Parents of a 9-year-old child came to thedentist complaining that their child presentswith enlarged cervical lymph nodes on the ri-ght. During examination palpation revealed thelymph nodes in the right submandibular, cervi-cal, supraclavicular, and infraclavicular areas tobe enlarged up to 2-2.5 cm in diameter, painless,non-matted together, and non-fused to the skin(resemble ”potatoes in a sack”). The parents noterapid fatigability and night sweats in their child.What additional examinations should the child bereferred for?

A. Puncture biopsy of the lymph nodesB. Pirquet and Mantoux testsC. Wassermann testD. CT of the cervical spineE. Clinical blood and urine tests